Sunteți pe pagina 1din 47

WEEK 2 In sum, there is no ambiguity in the terms of the contract and its riders.

INSURANCE Petitioner cannot rely on the general rule that insurance contracts are contracts
of adhesion which should be liberally construed in favor of the insured and
E. Essential Elements strictly against the insurer company which usually prepares it. A contract of
adhesion is xxx
GULF RESORTS INC vs PHILIPPINE CHARTER INSURANCE
CORPORATION (2005) We cannot apply the general rule on contracts of adhesion to the case at bar.
Petitioner cannot claim it did not know the provisions of the policy. From the
FACTS: Gulf Resorts, Inc at Agoo, La Union was insured with American Home inception of the policy, petitioner had required the respondent to copy verbatim
Assurance Company which includes loss or damage to shock to any of the the provisions and terms of its latest insurance policy from AHAC-AIU.
property insured by this Policy occasioned by or through or in consequence of
earthquake DOCTRINE:
It is basic that all the provisions of the insurance policy should be examined
July 16, 1990: an earthquake struck Central Luzon and Northern Luzon so the and interpreted in consonance with each other. All its parts are reflective of the
properties and 2 swimming pools in its Agoo Playa Resort were damaged true intent of the parties. The policy cannot be construed piecemeal. Certain
stipulations cannot be segregated and then made to control; neither do
August 23, 1990: Gulf's claim was denied on the ground that its insurance particular words or phrases necessarily determine its character.
policy only afforded earthquake shock coverage to the two swimming pools of
the resort Section 2(1) of the Insurance Code defines a contract of insurance as an
agreement whereby one undertakes for a consideration to indemnify another
Petitioner insists that the parties have intended to extend the coverage through against loss, damage or liability arising from an unknown or contingent event.
the attachment of the phrase "Subject to: Other Insurance Clause, Typhoon Thus, an insurance contract exists where the following elements concur:
Endorsement, Earthquake Shock Endorsement, Extended Coverage 1. The insured has an insurable interest;
Endorsement, FEA Warranty & Annual Payment Agreement on Long Term 2. The insured is subject to a risk of loss by the happening of the designated
Policies" to the insurance policy. peril;
3. The insurer assumes the risk;
ISSUE: Whether or not the insurance policy earthquake shock coverage 4. Such assumption of risk is part of a general scheme to distribute actual
extends to other property aside from the two swimming pools. losses among a large group of persons bearing a similar risk; and
5. In consideration of the insurer's promise, the insured pays a premium.
HELD: NO. Petitioner cannot focus on the earthquake shock endorsement to An insurance premium is the consideration paid an insurer for undertaking to
the exclusion of the other provisions. All the provisions and riders, taken and indemnify the insured against a specified peril. In fire, casualty, and marine
interpreted together, indubitably show the intention of the parties to extend insurance, the premium payable becomes a debt as soon as the risk attaches.
earthquake shock coverage to the two swimming pools only.
A contract of adhesion is one wherein a party, usually a corporation, prepares
A careful examination of the premium recapitulation will show that it is the the stipulations in the contract, while the other party merely affixes his
clear intent of the parties to extend earthquake shock coverage only to the two signature or his "adhesion" thereto. Consequently, any ambiguity therein is
swimming pools. resolved against the insurer, or construed liberally in favor of the insured.

In the subject policy, no premium payments were made with regard to PHILAMCARE HEALTH SYSTEMS, INC., vs. COURT OF APPEALS and
earthquake shock coverage, except on the two swimming pools. There is no JULITA TRINOS
mention of any premium payable for the other resort properties with regard to G.R. No. 125678 March 18, 2002
earthquake shock. This is consistent with the history of petitioner’s previous YNARES-SANTIAGO, J.:
insurance policies from AHAC-AIU.

Page 1 of 47
Facts:ErnaniTrinos, deceased husband of JulitaTrinos, applied for a health care 2. Defendants to pay the reduced amount of moral damages of P10,000.00 to
coverage withPhilamcare Health Systems, Inc. In the standard application plaintiff;
form, he answered “NO” to the following question: 3. Defendants to pay the reduced amount ofP10,000.00 as exemplary damages
to plaintiff;
Have you or any of your family members ever consulted or been treated for 4. Defendants to pay attorney’s fees of P20,000.00, plus costs of suit.
high blood pressure, heart trouble, diabetes, cancer, liver disease, asthma or
peptic ulcer? (If Yes, give details). CA: affirmed the decision of the trial court but deleted all awards for damages
and absolved petitioner Reverente.Denied MR.
Coverage of the health care agreement (HCA):
approved for a period of one year, Renewed 3 times yearly: March 1, 1988 - Issues:
March 1, 1990; March 1, 1990 – June 1, 1990. The amount of coverage was Whether health care agreements are considered insurance contracts.
increased to a maximum sum of P75,000.00 per disability. Whether there was concealment of material facts on the part of Ernani that
rendered the HCA void by virtue of the "Invalidation of agreement" contained
Ernani’s entitlement under HCA: in the contract.
hospitalization benefits, whether ordinary or emergency, listed therein Suppose there was concealment, what are the steps Philam should have done?
out-patient benefits" such as annual physical examinations, preventive health
care and other out-patient services. Ruling:
YES, it is an insurance contract.
Ernaniwas subsequently confined. HISTORY (everything happened within the Section 2 (1) of the Insurance Code defines a contract of insurance as an
period of coverage): agreement whereby one undertakes for a consideration to indemnify another
Ernani suffered a heart attack and was confined at the Manila Medical Center against loss, damage or liability arising from an unknown or contingent event.
(MMC) for one month beginning March 9, 1990. An insurance contract exists where the following elements concur:
Julita tried to claim the benefits under the health care agreement. (1) The insured has an insurable interest;
Philamdenied her claim saying that the Health Care Agreement was void. there (2) The insured is subject to a risk of loss by the happening of the designated
was a concealment regarding Ernani’s medical history. Doctors at the MMC peril;
allegedly discovered at the time of Ernani’s confinement that he was (3) The insurer assumes the risk;
hypertensive, diabetic and asthmatic, contrary to his answer in the application (4) Such assumption of risk is part of a general scheme to distribute actual
form. losses among a large group of persons bearing a similar risk; and
Julita paid the hospitalization expenses herself, amounting to about P76,000.00 (5) In consideration of the insurer’s promise, the insured pays a premium.
Ernani was discharged at MMC
He was attended by a physical therapist at home. Section 3 of the Insurance Code states that any contingent or unknown event,
Again he was admitted at the Chinese General Hospital. whether past or future, which may damnify a person having an insurable
Julita brought her husband home again due to financial difficulties. interest against him, may be insured against. Every person has an insurable
In the morning of April 13, 1990, Ernani had fever and was feeling very weak. interest in the life and health of himself. Section 10 provides:
Julita was constrained to bring him back to the Chinese General Hospital where
he died on the same day. Every person has an insurable interest in the life and health:
(1) of himself, of his spouse and of his children;
On July 24, 1990, respondent instituted with the Regional Trial Court of Manila, (2) of any person on whom he depends wholly or in part for education or
Branch 44, an action for damages against Philam and its president, Dr. Benito support, or in whom he has a pecuniary interest;
Reverente, She asked for reimbursement of her expenses plus moral damages (3) of any person under a legal obligation to him for the payment of money,
and attorney’s fees. After trial, the lower court ruled against Philam, ordered: respecting property or service, of which death or illness might delay or prevent
1. Defendants to pay and reimburse the medical and hospital coverage of the the performance; and
late ErnaniTrinos in the amount of P76,000.00 plus interest, until the amount is (4) of any person upon whose life any estate or interest vested in him depends.
fully paid to plaintiff who paid the same; In the case at bar, the insurable interest of respondent’s husband in obtaining
the health care agreement was his own health. The health care agreement was
Page 2 of 47
in the nature of non-life insurance, which is primarily a contract of indemnity. provider attaches once the member is hospitalized for the disease or injury
Once the member incurs hospital, medical or any other expense arising from covered by the agreement or whenever he avails of the covered benefits which
sickness, injury or other stipulated contingent, the health care provider must he has prepaid.
pay for the same to the extent agreed upon under the contract.
Philamshloud have followed Section 27 of the Insurance Code:
NONE, there was no concealment of material facts. "a concealment entitles the injured party to rescind a contract of insurance."
Petitioner cannot rely on the stipulation regarding "Invalidation of agreement" The right to rescind should be exercised previous to the commencement of an
which reads: action on the contract.In this case, no rescission was made. Besides, the
Failure to disclose or misrepresentation of any material information by the cancellation of health care agreements as in insurance policies require the
member in the application or medical examination, whether intentional or concurrence of the following conditions:
unintentional, shall automatically invalidate the Agreement from the very Prior notice of cancellation to insured;
beginning and liability of Philamcare shall be limited to return of all Membership Notice must be based on the occurrence after effective date of the policy of one
Fees paid. An undisclosed or misrepresented information is deemed material if or more of the grounds mentioned;
its revelation would have resulted in the declination of the applicant by Must be in writing, mailed or delivered to the insured at the address shown in
Philamcare or the assessment of a higher Membership Fee for the benefit or the policy;
benefits applied for. Must state the grounds relied upon provided in Section 64 of the Insurance
Code and upon request of insured, to furnish facts on which cancellation is
The answer assailed by petitioner was in response to the question relating to based.
the medical history of the applicant. This largely depends on opinion rather None of the above pre-conditions was fulfilled in this case.
than fact, especially coming from respondent’s husband who was not a medical
doctor. Where matters of opinion or judgment are called for, answers made in Anent the incontestability of the membership of respondent’s husband, we
good faith and without intent to deceive will not avoid a policy even though quote with approval the following findings of the trial court:
they are untrue. Thus,
(U)nder the title Claim procedures of expenses, the defendant Philamcare
(A)lthough false, a representation of the expectation, intention, belief, opinion, Health Systems Inc. had twelve months from the date of issuance of the
or judgment of the insured will not avoid the policy if there is no actual fraud in Agreement within which to contest the membership of the patient if he had
inducing the acceptance of the risk, or its acceptance at a lower rate of previous ailment of asthma, and six months from the issuance of the
premium, and this is likewise the rule although the statement is material to the agreement if the patient was sick of diabetes or hypertension. The periods
risk, if the statement is obviously of the foregoing character, since in such case having expired, the defense of concealment or misrepresentation no longer lie.
the insurer is not justified in relying upon such statement, but is obligated to
make further inquiry. There is a clear distinction between such a case and one
in which the insured is fraudulently and intentionally states to be true, as a F. Applicability of Civil Code
matter of expectation or belief, that which he then knows, to be actually
untrue, or the impossibility of which is shown by the facts within his Art. 2011. The contract of insurance is governed by special laws. Matters not
knowledge, since in such case the intent to deceive the insurer is obvious and expressly provided for in such special laws shall be regulated by this Code. (n)
amounts to actual fraud. (Underscoring ours)
Art. 2012. Any person who is forbidden from receiving any donation under
The fraudulent intent on the part of the insured must be established to warrant Article 739 cannot be named beneficiary of a life insurance policy by the person
rescission of the insurance contract. Concealment as a defense for the health who cannot make any donation to him, according to said article. (n)
care provider or insurer to avoid liability is an affirmative defense and the duty
to establish such defense by satisfactory and convincing evidence rests upon Art. 739. The following donations shall be void:
the provider or insurer. In any case, with or without the authority to
investigate, petitioner is liable for claims made under the contract. Having (1) Those made between persons who were guilty of adultery or concubinage
assumed a responsibility under the agreement, petitioner is bound to answer at the time of the donation;
the same to the extent agreed upon. In the end, the liability of the health care
Page 3 of 47
(2) Those made between persons found guilty of the same criminal offense, in contracts. And under Article 2012 of the same Code, any person who is
consideration thereof; forbidden from receiving any donation under Article 739 cannot be
(3) Those made to a public officer or his wife, descedants and ascendants, by named beneficiary of a fife insurance policy by the person who cannot make a
reason of his office. donation to him. Common-law spouses are barred from
In the case referred to in No. 1, the action for declaration of nullity may be receiving donations from each other.
brought by the spouse of the donor or donee; and the guilt of the donor and Article 739 provides that void donations are those made between persons who
donee may be proved by preponderance of evidence in the same action. (n) were guilty of adultery or concubinage at the time of donation.
There is every reason to hold that the bar in donations between legitimate
spouses and those between illegitimate ones should be enforced in life
INSULAR V EBRADO G.R. NO. L-44059 OCTOBER 28, 1977 insurance policies since the same are based on similar consideration. So long
as marriage remains the threshold of family laws, reason and
Facts: morality dictate that the impediments imposed upon married couple should
J. Martin: likewise be imposed upon extra-marital relationship.
Cristor Ebrado was issued by The Life Assurance Co., Ltd., a policy for A conviction for adultery or concubinage isn’t required exacted before
P5,882.00 with a rider for Accidental Death. He designated Carponia T. Ebrado the disabilities mentioned in Article 739 may effectuate. The article says that in
as the revocable beneficiary in his policy. He referred to her as his wife. the case referred to in No. 1, the action for declaration of nullity may be
Cristor was killed when he was hit by a failing branch of a tree. Insular Life was brought by the spouse of the donor or donee; and the guilty of the donee may
made liable to pay the coverage in the total amount of P11,745.73, be proved by preponderance of evidence in the same action.
representing the face value of the policy in the amount of P5,882.00 plus the The underscored clause neatly conveys that no criminal conviction for the
additional benefits for accidental death. offense is a condition precedent. The law plainly states that the guilt of the
Carponia T. Ebrado filed with the insurer a claim for the proceeds as the party may be proved “in the same acting for declaration of nullity of donation.”
designated beneficiary therein, although she admited that she and the insured And, it would be sufficient if evidence preponderates.
were merely living as husband and wife without the benefit of marriage. The insured was married to Pascuala Ebrado with whom she has six legitimate
Pascuala Vda. de Ebrado also filed her claim as the widow of the deceased children. He was also living in with his common-law wife with whom he has two
insured. She asserts that she is the one entitled to the insurance proceeds. children.
Insular commenced an action for Interpleader before the trial court as to who
should be given the proceeds. The court declared Carponia as disqualified. ZENITH INSURANCE CORPORATION V. THE INSURANCE COMMISSION-
INSURABLE INTEREST
Issue: WON a common-law wife named as beneficiary in the life insurance
policy of a legally married man can claim the proceeds in case of death of the Facts:
latter? > Zenith entered into an insurance contract, denominated as Equipment
Floater Policy covering a Kato Bachoe including its accessories and
Held: No. Petition appurtenances thereof, from loss of damage. Complainant paid the stipulated
premiums therefore.
Ratio: > Within the period of effectivity of the policy, the two pieces of hydraulic
Section 50 of the Insurance Act which provides that "the insurance shall be wheel gear pumps, which are considered appurtenances and/or parts attached
applied exclusively to the proper interest of the person in whose name it is to and/or installed in the Kato BAchoe were lost, stolen and/or illegally
made" detached by unknown thieves or malefactors
The word "interest" highly suggests that the provision refers only to the > Despite repeated assurances by Zenith’s soliciting agent, it refused and
"insured" and not to the beneficiary, since a contract of insurance is personal in failed to settle and pay complainant’s insurance claim.
character. Otherwise, the prohibitory laws against illicit relationships especially > Complainant seeks not only the payment of said insurance claim of 70T plus
on property and descent will be rendered nugatory, as the same could easily be legal interest, atty’s fees, and litigation expenses, but also the revocation or
circumvented by modes of insurance. cancellation of the license of Zenith to do insurance business.
When not otherwise specifically provided for by the Insurance Law, the contract > Zenith on the other hand contends that:
of life insurance is governed by the general rules of the civil law regulating
Page 4 of 47
o Complainant is not the real party in interest since the policy carries with it endorsement, it cannot now allege that complainant has no insurable interest
a designated loss payee, the BA Finance Corp on the property insured. Zenith is now precluded by the equitable principle of
o The policy insures against loss or damage caused by fire and lightning, etc, estoppel from impugning and dishonoring the very insurance policy contract it
while theft or robbery is NOT insured against in the policy, it not having been issued and the endorsement and increase in the coverage made through its
expressly mentioned duly authorized agent.
o Loss nevertheless is excluded under the exception of “infidelity exclusion”
by the operator who left it unguarded, unattended and deserted while G. Subrogation
entrusted to him, and for failure to give timely notice of loss
o Complainant and/or BA Finance is guilty of concealment and Art. 2027. No annuity shall be claimed without first proving the existence of the
misrepresentation at the time they secured the policy, because at the time it person upon whose life the annuity is constituted.
became operative, the complainant was NOT yet the owner of the property
insured, the property still hot having been delivered to him, and BA finance had FIREMAN’S FUND INSURANCE COMPANY and FIRESTONE TIRE AND RU
no insurable interest yet, henceforth, the contract of insurance was VOID AB BBER COMPANY OF THE PHILIPPINES vs. JAMILA & COMPANY, INC.
INITIO for lack of insurable interest at the time the insurance took effect. and FIRST QUEZON CITY INSURANCE CO., INC

Issues and Resolutions: SUMMARY: Jamila supplies security guards to Firestone and assumes their
(1) Whether or not the loss through theft or robbery claimed is within the responsibility. When some properties of Firestone were lost due to connivance
coverage of the policy. of some security guards, Fireman’s Fund as insurer paid Firestone the value of
The Insurance Commissioner, as reiterated by the SC, found for the such and is now subrogated to Firestone’s right to reimbursement. They filed
complainant in this wise: While the policy enumerated the risks covered, it complaint to recover money when Jamila failed to pay. CFI dismissed complaint
does NOT, however, in its express terms, limit compensability to that stated in as to Jamila citing that there is no cause of action as the latter did not consent
the enumeration. The enumerated risks excluded did not include theft or to subrogation and there are no allegations in the complaint that Firestone
robbery committed or perpetrated by an unidentified culprit, hence the investigated the loss. Subsequent MRs, F&F argue that their cause of action is
complainant’s claim for damages is compensable. on the basis of legal subrogation.
SC: There was cause of action on the part of Fireman’s Fund pursuant to Art.
The foregoing policy is supported by the long time honored doctrine of “contra 2207. Payment by the assurer to the assured operates as an equitable
proferentem: which provides that: “any ambiguity in the policy shall be assignment to the assurer of all the remedies which the assured may have
resolved in favor of the insured and against the insurer”. This is true because against the third party whose negligence or wrongful act caused the loss.
insurance contracts are essentially contracts of adhesion and applicants for DOCTRINE: Loss or injury for risk must be covered by the policy – Under
insurance have no choice but to accept the terms and conditions in the policy Article 2207, the cause of the loss or injury must be a risk covered by the
even if they are not in full accord therewith. policy to entitle the insurer to the subrogation. Thus, where the insurer pays
the insured for a loss which is not a risk covered by the policy, thereby
(2) Whether or not the complainant was with insurable interest therein when effecting “voluntary payment,” the insurer has no right of subrogation against
the said policy contract was procured. the third party liable for the loss. Nevertheless, the insurer may recover from
The complainant has insurable interest in the insured property at the time of the third party responsible for the damage to the insured property under Article
the procurement of the insurance policy. As the CC provides, “the contract of 1236 of the Civil Code.
sale is perfected at the moment there is a meeting of minds upon the thing
which is the object of the contract and upon the price,” and Sec. 15 of the IC FACTS:
allows the insurance of a mere contingent or expectant interest in anything if Jamila or the Veterans Philippine Scouts Security Agency contracted to supply
the same is founded on an actual right to the thing, or upon any valid contract. security guards to Firestone. Jamila assumed responsibility for the acts of its
security guards
As this is the case, mere possession of an equitable title, like that pertaining to First Quezon City Insurance Co., Inc. executed a bond in the sum of P20k to
the buyer, gives rise to insurable interest in the property in which such title guarantee Jamila’s obligations under that contract
inheres. Furthermore, considering that Zenith’s agent had been fully apprised May 18, 1963: Properties of Firestone valued at Php 11,925 were lost allegedly
of the circumstances prior to the actual issuance of the policy and the due to the acts of its employees who connived with Jamila’s security guard

Page 5 of 47
Fireman’s Fund, as insurer, paid to Firestone the amount of the loss and is injury or loss arising out of the wrong or breach of contract complained of, the
now subrogated to Firestone’s right to get reimbursement from Jamila insurance company shall be subrogated to the rights of the insured
Jamila and its surety, First Quezon City Insurance Co., Inc., failed to pay the against the wrongdoer or the person who has violated the contract”.
amount of the loss in spite of repeated demands. CFI on F&F MR: Denied motion
Fireman’s Fund and Firestone Tire and Rubber Co instituted this complaint F&F filed 2nd MR and called CFI’s attention to the fact that the issue of
against Jamila for the recovery of the sum of Php 11,925.00 plus interest, subrogation was of no moment because Firestone, the subrogor (??), is a
damages and attorney’s fees party-plaintiff and could sue directly Jamila in its own right.
Jamila moved to dismiss the complaint on the ground of lack of cause of action CFI on F&F’S 2nd MR:Denied 2nd MR without resolving contention
(1) complaint did not allege that Firestone, pursuant to the contractual Appeal to SC
stipulation quoted in the complaint, had investigated the loss and that Jamila F&F: CFI’s dismissal of their complaint is contrary to Article 2207 which
was represented in the investigation and provides for legal subrogation.
(2) Jamila did not consent to the subrogation of Fireman’s Fund to JAMILA: Legal subrogation under Art. 2207 requires the debtor’s consent
Firestone’s right to get reimbursement from Jamilaand its surety. o Legal subrogation takes place in the cases mentioned in NCC 1302 and the
CFI: Dismissed the complaint as to Jamila on the second ground that there instant case is not among the 3 cases enumerated in that article
wasno allegation that it had consented to the subrogation and, therefore, o There could be no subrogation in this case because according to F&F, the
Fireman’s Fund had no cause of action against it. contract between Jamila and Firestone was entered into on June
Also dismissed the complaint as to First Quezon City Insurance Co., Inc. on the 1, 1965 but the loss complained of occurred on May 18, 1963.
ground of res judicata as the same action was previously filed in a civil case
which was dismissed because of the failure of the same plaintiffs and their ISSUES:
counsel to appear at the pre-trial. 1) Whether the complaint of Firestone as subrogor (???) states a cause of
Firestone and Fireman’s Fund filed MR action against Jamila? (Not really)
CFI on F&F’s MR: Set aside its order of dismissal. 2) Whether the complaint of Fireman’s Fund as subrogee states a cause of
No res judicata as to First Quezon City Insurance Co., because civil case action against Jamila? (YES)
was dismissed without prejudice 3) Whether Jamila should reimburse Fireman’s Fund? (Not decided here)
However, due to inadvertence, the lower court did not state in its order of
September 3, 1966 why it set aside its prior order dismissing the HELD:
complaint with respect to Jamila. CFI Decision’s order of dismissal is legally untenable so SET ASIDE with costs
First Quezon City Insurance Co., Inc. filed its answer to the complaint. against Jamila & Co., Inc.
Jamila, upon noticing that the order had obliterated its victory without any RATIO:
reason therefor, filed MR reconsideration [F&F’s counsel gratuitously alleged in their brief that Firestone and Jamila
Invoked the first ground in its original motion to dismiss which had never entered into a “contract of guard services” on June 1, ‘65.That allegation was
been passed upon by the lower court that complaint did not allege that uncalled for because it is not found in the complaint and so created confusion
Firestone, pursuant to the contractual stipulation quoted in the complaint, which did not exist. No copy of the contract was annexed to the complaint.
had investigated the loss and that Jamila was represented in the That confusing statement was an obvious error since it was expressly alleged in
investigation the complaint that the loss occurred on May 18, ‘63. The fact that such an
CFI on Jamila’s MR: Granted Jamila’s MR. However, it completely ignored the error was committed is another instance substantiating the observation
1st ground but reverted to the second ground (no consent to subrogation thus that F&F’s counsel had not exercised due care in the presentation of his case.]
no cause of action). 1) Firestone is really a nominal party in this case as it had already been
It did not mention Firestone, the co-plaintiff of Fireman’s Fund. indemnified for the loss which it had sustained. It joined as a party-plaintiff in
Firestone and Fireman’s Fund filed MR on the ground that Fireman’s Fund order to help Fireman’s Fund to recover the amount of the loss from Jamila and
Insurance Company was suing on the basis of legal subrogation whereas First Quezon City Insurance Co., Inc. Firestone had tacitly assigned to
CFI erroneously predicated its dismissal order on the theory that there was no Fireman’s Fund its cause of action against Jamila for breach of contract.
conventional subrogation because the debtor’s consent was lacking. Sufficient ultimate facts are alleged in the complaint to sustain that cause of
Cited NCC 2207 which provides that “if the plaintiff’s property has been action.
insured, and he has received indemnity from the insurance company for the
Page 6 of 47
2) Fireman’s Fund’s action against Jamila is squarely sanctioned by article PAN MALAYAN INSURANCE CORPORATION, petitioner, vs. COURT OF
2207. As the insurer, Fireman’s Fund is entitled to go after the person or entity APPEALS, ERLINDA FABIE AND HER UNKNOWN DRIVER,
that violated its contractual commitment to answer for the loss insured
against (PAL vs. Heald Lumber Co).
FACTS:
CFI erred in applying to this case the rules on novation. F&F in alleging in their
1. Petitioner Panmalay was an insurer of the car of CANLUBANG AUTOMOTIVE
complaint that Fireman’s Fund “became a party ininterest in this case by virtue
RESOURCE CORP. which was bumpt and damaged by the private respondent
of a subrogation right given in its favor by” Firestone, were not relying on the
through its negligent driver.
novation by changeof creditors as contemplated in NCC 1291 and 1300 to
2. Petitioner PANMALAy paid the amount of insurance to the insured.
1303 but rather on NCC 2207.
3. Subrogated on the rights of the insured, petitioner demand payment from
the private respondent who refused to pay the claim of the petitioner.
Article 2207 is a restatement of a settled principle of American jurisprudence.
4. Petitioner filed a complaint against private respondent before the RTC.
Subrogation has been referred to as the doctrine of substitution. It “is an arm
5. Private respondent filed a motion to dismiss arguing that payment under the
of EQUITY that may guide or even force one to pay a debt for which
“own damage” clause of the insurance policy precluded subrogation under
an obligation was incurred but whichwas in whole or in part paid by another”
Article 2207 of the Civil Code, since indemnification thereunder was made on
(83 C.J.S. 576).
the assumption that there was no wrongdoer or no third party at fault.
“Subrogation is founded on principles of JUSTICE AND EQUITY, and its
6. The RTC dismissed the complaint aswell as the motion for reconsideration
operation is governed by principles of equity. It rests onthe principle that
and this was affirmed by the CA.
substantial justice should be attained regardless of form, that is, its basis is the
doing of complete, essential, andperfect justice between all the parties
ISSUE:
without regard to form”(83 C.J.S. 579- 80)
WHETHER OR NOT, THE PETITIONER IS ALLOWED TO RECOVERED THE
Subrogation is a normal incident of indemnity insurance (Aetna L. Ins. Co. vs
AMOUNT OF INSURANCE IT HAD PAID TO THE INSURED FROM PRIVATE
Moses). Upon payment of the loss, the insurer is entitled to be
RESPONDENT.
subrogate pro tanto to any right of action which the insured may have
against the third person whosenegligence or wrongful act caused the
RULING:
loss (44 Am. Jur. 2nd 745).
According to the Supreme Court, Art. 2207 of the Civil Code states that, “If the
plaintiffs property has been insured, and he has received indemnity from the
The right of subrogation is of the highest EQUITY. The LOSS IN THE FIRST
insurance company for the injury or loss arising out of the wrong or breach of
INSTANCE is that of the INSURED but AFTERreimbursement or compensation,
contract complained of, the insurance company shall be subrogated to the
it becomes the LOSS OF THE INSURER (44 Am. Jur. 2d 746).
rights of the insured against the wrongdoer or the person who has violated the
“Although many policies including policies in the standard form, now provide
contract.”
for subrogation, and thus determine the rights of theinsurer in this respect, the
This was founded on the well-settled principle of subrogation. If the insured
equitable right of subrogation as the legal effect of payment inures to the
property is destroyed or damaged through the fault or negligence of a party
insurer without any formalassignment or any express stipulation to that
other than the assured, the insurer, upon payment to the assured, will be
effect in the policy” (44 Am. Jur. 2nd 746).
subrogated to the rights of the assured to recover from the wrongdoer to the
extent that the insurer has been obligated to pay. Payment by the insurer to
Stated otherwise, when the insurance company pays for the loss,
the assured operates as an equitable assignment to the former of all remedies
such payment operates as an equitable assignment to the insurer of the
which the latter may have against the third party whose negligence or wrongful
property and all remedies which the insured may have for the recovery
act caused the loss.
thereof. That right is not dependent upon, nor does it grow out of, any privity
WHEREFORE, in view of the foregoing, the present petition is GRANTED.
of contract, or upon written assignment of claim, and payment to the insured
Petitioner’s complaint for damages against private respondents is reinstated.
makes the insurer an assignee in equity (Shambley v. Jobe-Blackley Plumbing
So the case was remanded to the Trial Court for the trial of the merit.
and Heating Co).3) Whether the plaintiffs would be able to prove their cause of
In the pertinent case of Sveriges Angfartygs Assurans Forening v. Qua Chee
action against Jamila is another question.
Gan, supra.,the Court ruled that the insurer who may have no rights of
subrogation due to “voluntary” payment may nevertheless recover from the

Page 7 of 47
third party responsible for the damage to the insured property under Article (b) that plaintiff's insurance policy did not cover the shortshipment and (c)
1236 of the Civil Code. defendant was merely acting as an agent of Louis Dreyfus & Co., who was the
In the pertinent case of Sveriges Angfartygs Assurans Forening v. Qua real shipper.
Chee Gan, supra., the Court ruled that the insurer who may have no
rights of subrogation due to “voluntary” payment may nevertheless Taking issue with all the foregoing, plaintiff has interposed the present appeal
recover from the third party responsible for the damage to the insured to Us on questions of fact and law, the amount involved exceeding
property under Article 1236 of the Civil Code. P200,000.00.

SVERIGES ANFARTYGS ASSURANCE VS QUA CHEE GAN (full case) Issue:

Was the non-presentation of the insurance policy fatal to plaintiff's case? The
On August 23 and 24, 1947, defendant Qua Chee Gan, a sole proprietorship,
lower court ruled so, reasoning that unless the same as the best evidence were
shipped on board the S.S. NAGARA as per bills of lading Exhs. A and B
presented, it could not be conclusively determined if "liability for short
2,032,000 kilos of bulk copra at Siain, Quezon, consigned to DAL International
shipment" was a covered risk. And the rule is that an insurer who pays the
Trading Co., in Gdynia, Poland. The vessel first called at the port
insured for loss or liability not covered by the policy is not subrogated to the
of Karlshamn, Sweden, where it unloaded 696,419 kilos of bulk copra. Then, it
latter.[2] However, even assuming that there was unwarranted - or "volunteer"
proceeded to Gdynia where it unloaded the remaining copra shipment. The
- payment, plaintiff could still recover what it paid - in effect - to the carrier
actual outturn weights in the latter port showed that only 1,569,429 kilos were
from defendants shipper under Art. 1236 of the Civil Code which allows a third
discharged.
person who pays on behalf of another to recover from the latter, although
Because of the alleged confirmed cargo shortage, the Polish cargo insurers had there is no subrogation. But since the payment here was without the
to indemnify the consignee for the value thereof. Thereafter, the former sued knowledge and consent of defendant, plaintiff's right of recovery
the ship-owner, the Swedish East Asia Company, in Gothenburg, Swe- is defeasible by the former's defenses since the Code is clear that the recovery
den. The latter, in turn, sued defendant and had it summoned to only up to the amount by which the defendant was benefited.
Gothenburg. Defendant However refused to submit to that court's jurisdiction
This brings Us to the crux of them case: Was there a shortshipment? To
and its objection was sustained.
support its case, plaintiff theorizes that defendant had two
In March, 1951, a settlement was effected between the Polish cargo insurers shipments at Siain, Quezon province: (1) 812,800 kilos for Karlshamn and (2)
and the ship owner. Plaintiff, as the indemnity insurer for the latter, paid 2,032,000 kilos for Gdynia. The Karlshamn shipment was asserted to have
approximately $60,733.53 to the Polish insurers. On August 16, 1954, been covered by a separate bill of lading which however was allegedly lost
claiming to have been subrogated to the rights of the carrier, plaintiff sued subsequently. Thus, the 696,419 kilos of copra unloaded in Karlshamn was not
defendant before the Court of First Instance of Manila to part of the Gdynia shipment and cannot explain the confirmed shortage at the
recover U.S. $60,733.53 plus 17% exchange tax, with legal interest, as the latter port.
value of the alleged cargo short shipment and P10,000 as attorney's
Plaintiff's cause of action suffers from several fatal defects and
fees. Defendant answered in due time and countered with a
inconsistencies. The alleged shipment of 812,800 kilos for Karlshamn is
P15,000 counterclaim for attorney's fees.
contradicted by plaintiff's admission in paragraphs 2 and 3 of its complaint that
On August 1, 1955, defendant filed a motion to dismiss on the ground of defendant shipped only 2,032,000 kilos copra
prescription under the Carriage of Goods by Sea Act. The lowercourt sustained at Siain, purportedly for both Gdynia and Karlshamn.[3] Needless to state,
the motion and plaintiff appealed here. We reversed the order of dismissals plaintiff is bound by such judicial admission.[4] Moreover, the alleged existence
and remanded the case for further proceedings.[1] of the Karlshamn bills of lading is negative by the fact that Exhibits A and B -
the bills of lading presented by plaintiff - show that the 2,032,000 kilos of
After trial the lower court September 28, 1963, rendered its decisions copra loaded in Siain were for Gdynia only. Further destroying its case is the
dismissing the complaint and awarding P10,000 as attorney's fees to testimony of plaintiff's own witness, Mr. Claro Pasicolan, who on direct
defendant. It ruled (a) that there was no shortshipment on defendant's part examination affirmed[5] that these two exhibits constituted the complete set of
Page 8 of 47
documents which them shipping agent in charge of the vessel RIZAL SURETY AND INSURANCE CO. vs. MANILA RAILROAD CO. AND
S.S. NAGARA issued covering the copra cargo loaded at Slain. In view of MANILA PORT SERVICE
this admission and for want of evidentiary support, plaintiff's belated claim that
there is another complete set of documents can not be seriously taken. Facts:
On Nov 29, 1960, a vessel named SS Flying Trader, loaded on board a cargo
Lastly, if there really was a separate bill of lading for the Karlshamn shipment,
which is an offset press machine, from Italy to Manila. Upon reaching the port
plaintiff could not have failed to present a copy thereof. Mr. Pasicolan testi-
of destination and upon unloading it, it was dropped b the crane which resulted
fied[6] that the shipping agent makes 20 copies of the documents of which to damages of the machine. The plaintiff as the insurer had paid the consignee,
three signed ones are given to the shipper and the rest, marked as non- Suter, Inc. the amount of P16.5k for the machine and P180.70 for the
negotiable bills of lading - like Exhibits A and B - are kept on its file. For the International Adjustment Bureau as adjuster’s fee. However, the arrastre
three signed copies to be lost,We may believe, but not for all the remaining 17 charges in this particular shipment was paid on the weight or measurement
other copies. Under the circumstances, it his more reasonable to hold that basis whichever is higher, and not on the value thereof.
there was no separate shipment intended for Karlshamn, Sweden.
Issue:
As a corollary to the foregoing conclusion, it stands to reason that the copra Can the insurance get an amount greater than what was declared?
unloaded in Karlshamn formed part of the same - and only - shipment of
defendant intended for Gdynia. Now the fact that the sum total of the cargo Held:
unloaded at Karlshamn and Gdynia would exceed what appears to have been Plaintiff Insurance Company cannot recover from defendants an amount
loaded at Sian by as much as 233,848 kilos can only show that defendant greater than that to which the consignee could lawfully lay claim. The
really overshipped, not shortshipped. And while this would not tally with management contract is clear, the amount is limited to P500.
defendant's claim of having weighed the copra cargo 100% at Siain, thus If the plaintiff’s property has been insured, and he has received indemnity from
exposing a flaw in defendant's case, yet it is elementary that plaintiff must rely the insurance company for the injury or loss arising out of the wrong or breach
of contract complained of, the insurance company shall be subrogated to the
on the strength of it own case to recover, and not bank on the weakness of the
right of the insured against the wrong-doer or the person who has violated the
defense. Plaintiff here failed to establish its case by preponderance on
contract. If the amount paid by the insurance company doer not fully cover the
evidence.
injury or loss, the aggrieved party shall be entitled to recover the deficiency
On the question whether defendant is the real shipped or merely an agent of from the person causing the loss or injury.
Louis Dreyfus & Co., suffice it to say that although on Exhibit A and B his name The insurance have no greater right than the party in interest thereof.
appears as the shipper, yet the very loading certificate, Exhibit 3 [5-Deposition
PAUL FIRE & MARINE INSURANCE v MACONDRAY & CO
of Horle], issued and signed by the Chief Mate, and Master of the S.S. NAGARA
shows that defendant was acting merely for account of Louis Dreyfus & Facts:
Co. The other documentary exhibits[7] confirm this. Anyway, in whatever -Winthrop Products, Inc., of New York shipped aboard the SS “Tai Ping”, owned
capacity defendant is considered, it cannot be liable since and operated by Wilhelm Wilhelmsen218 cartons and drums of drugs and
no shortshipment was shown. medicine with Winthrop-Stearns Inc., Manila, Philippines as consingee.
BarberSteamship Lines, Inc., agent of Wilhelm Wilhelmsen issued Bill of Lading
Plaintiff's action against defendant cannot, however, be considered as clearly No. 34, in the name of Winthrop Products.
unfounded as to warrant an award of attorney's fees as damages to defendant -The shipment was insured by the shipper against loss and/or damage with the
under par. 4, Art. 2208 of the Civil Code. The facts do not show that plaintiff's St. Paul Fire & Marine InsuranceCompany.
cause of action was so frivolous or untenably as to amount to -“Tai Ping” arrived at the Port of Manila.
gross and evident bad faith.[8] -The said shipment was discharged complete and in good order with the
exception of one (1) drum and several cartonswhich were in bad condition.
WHEREFORE, but for the award of attorney's fees to defendant which is -Because consignee failed to receive the whole shipment and as several cartons
eliminated, the decision appealed from is, in all other respects, hereby of medicine were received in badorder condition, Winthrop-Sterns Philippines
affirmed. Costs against plaintiff-appellant.
Page 9 of 47
filed the corresponding claim in the amount of Pl,109.67 representingthe C.I.F. -The stipulation in the bill of lading limiting the common carrier’s liability to the
value of the damaged drum and cartons of medicine with the carrier and the value of the goods appearing in thebill, unless the shipper or owner declares a
arrestre. greater value, is valid and binding.
-However, both refused to pay. -This limitation of the carrier’s liability is sanctioned by the freedom of the
-Winthrop-Sterns Philippines filed its claim with the insurer, St. Paul Fire & contracting parties to establish suchstipulations, clauses, terms, or conditions
Marine insurance. as they may deem convenient, provided they are not contrary to law,morals,
-The insurance company, on the basis of such claim, paid to the consignee the good customs and public policy.
insured value of the lost and damagedgoods, including other expenses in -A stipulation fixing or limiting the sum that may be recovered from the carrier
connection therewith, in the total amount of $1,134.46. on the loss or deterioration of the goods is valid, provided it is:
-As subrogee of the rights of the shipper and/or consignee, the insurer, St. (a) reasonable and just under the circumstances, and
Paul Fire & Marine Insurance Co., instituted with the Court of First Instance the (b) has been fairly and freely agreed upon.
present action against the defendants for the recovery of said amount -In the case at bar, the liabilities of the defendants- appellees with respect to
of $1,134.46, plus costs. the lost or damaged shipments areexpressly limited to the C.I.F. value of the
-The Lower court rendered judgment ordering defendants Macondray & Co., goods as per contract of sea carriage embodied in the bill of lading,
Inc., Barber Steamship Lines, Inc. andWilhelm Wilhelmsen to pay to whichreads:
the plaintiff P300.00. It also held defendants Manila Railroad Company o Whenever the value of the goods is less than $500 per package or other
and Manila PortService to pay to plaintiff, jointly and severally, the sum freight unit, their value in the calculation and adjustment of claims for which
of P809.67. the Carrier may be liable shall for the purpose of avoiding uncertainties and
-The Insurer, , contending that it should recover the amount of $1,134.46 or difficulties in fixing value be deemed to be the invoice value, plus freight and
its equivalent in pesos (the rate of P3.90,instead of P2.00, for every US$1.00), insurance if paid, irrespective of whether any other value is greater or less.
filed a motion for reconsideration, but this was denied.
-The Insurer argues that, as subrogee of the consignee, it should be entitled to NATIONAL UNION FIRE INSURANCE VS. STOLT NIELSEN
recover from the defendants-appelleesthe amount of $1,134.46 which it
actually paid to the consignee and which represents the value of the lost EMERGENCY RECIT: United Coconut Chemicals (SHIPPER) shipped distilled
anddamaged shipment as well as other legitimate expenses such as the duties fatty acid on board MT “StoltSceptre” (CARRIER). The shipment was insured
and cost of survey of said shipment, andthat the exchange rate on the date under a marine cargo policy with National Union Fire Insurance Co (INSURER).
of the judgment, which was P3.90 for every US$1.00. Upon receipt of the cargo by the consignee in Netherlands, it was totally
-Defendants-appellees countered that: contaminated. Hence, claim was made on the INSURER of the cargo. The
o Their liability is limited to the C.I.F. value of the goods, pursuant to contract INSURER as subrogee filed a claim for damages against the CARRIER with RTC
of sea carriage embodied in the bill of lading that the consignee’s (Winthrop-
Manila. The CARRIER invoked that arbitration must be done pursuant to the
Stearns Inc.) claim against the carrier (Macondray & Co., Inc.,Barber
Charter. The INSURER opposed, arguing that the provision on arbitration was
Steamship Lines, Inc., Wilhelm Wilhelmsen and the arrastre operators (Manila
Port Service and Manila Railroad Company) was only for the sum of Pl,109.67 not included in the Bill of Lading. SC: The INSURER cannot avoid the binding
effect of the arbitration clause. By subrogation, it became privy to the Charter
ISSUE(S): Party as fully as the SHIPPER before the latter was indemnified, because as
1.Whether or not, in case of loss or damage, the liability of the carrier to the subrogee it stepped into the shoes of the SHIPPER and is subrogated merely to
consignee is limited to the C.I.F value of the goods which were lost or damaged the latter's rights.
2.Whether the insurer who has paid the claim in dollars to the consignee should
be reimbursed in its peso equivalent on the date of discharge of the cargo or on FACTS:
the date of the decision.
 On 9 January 1985, United Coconut Chemicals, Inc. shipped 404.774
metric tons of distilled C6-C18 fatty acid on board MT "Stolt Sceptre," a
HELD:
tanker owned by Stolt-Nielsen Philippines Inc., from Bauan, Batangas,
The appeal is without merit and the judgement of the lower court is affirmed.
Philippines, consigned to "Nieuwe Matex" at Rotterdam, Netherlands,
-The purpose of the bill of lading is to provide for the rights and liabilities of the
covered by Tanker Bill of Lading BL No. BAT-1.
parties in reference to the contract tocarry.
Page 10 of 47
 The shipment was insured under a marine cargo policy with Petitioner the respondent Appellate Court found, the INSURER "cannot feign ignorance of
National Union Fire Insurance Company of Pittsburg (hereinafter referred the arbitration clause since it was already charged with notice of the existence
to as INSURER), a non-life American insurance corporation, through its of the charter party due to an appropriate reference thereof in the bill of lading
settling agent in the Philippines, the American International Underwriters and, by the exercise of ordinary diligence, it could have easily obtained a copy
(Philippines), Inc., the other petitioner herein. thereof either from the shipper or the charterer.
 Upon receipt of the cargo by the consignee in the Netherlands, it was
found to be discoloured and totally contaminated. Hence, a claim was We hold, therefore, that the INSURER cannot avoid the binding effect of the
made on the Insurer of the cargo. The insurer as subrogee filed a claim arbitration clause. By subrogation, it became privy to the Charter Party as fully
for damages against the carrier with the RTC of Manila. as the SHIPPER before the latter was indemnified, because as subrogee it
 The carrier filed a motion to dismiss on the ground that the case was stepped into the shoes of the SHIPPER-ASSURED and is subrogated merely to
arbritrable and pursuant to the charter party as embodied in the bill of the latter's rights. It can recover only the amount that is recoverable by the
lading, arbitration must be done. The insurer opposed the motion by
assured. And since the right of action of the SHIPPER-ASSURED is governed by
arguing that the provision on arbitration was not included in the bill of
the provisions of the Bill of Lading, which includes by reference the terms of the
lading and even if it was included, it was nevertheless unjust and
unreasonable. Charter Party, necessarily, a suit by the INSURER is subject to the same
 The RTC denied the motion but upon reconsideration, the resolution on agreements. It has not been shown that the arbitral clause in question is null
the motion to dismiss was suspended or deferred. and void, inoperative, or incapable of being performed. Nor has any conflict been
 The carrier then filed a petition for review on certiorari with preliminary pointed out between the Charter Party and the Bill of Lading.
injunction/TRO which was granted by the CA.
CEBU SHIPYARD V WILLIAM LINES
ISSUE: Are the terms of the Charter Party, particularly the provision on
Nature of Case: Petition for Review on Certiorari
arbitration, binding on the INSURER?
Digest maker: Erika Potian
HELD: Yes. The pertinent portion of the Bill of Lading in issue provides in part:
SUMMARY: William Lines insured its M/V Manila City under
xxx [A]ll the terms whatsoever of the said Charter except the rate and
Prudential Guarantee for hull and machinery. Policy contained clause
payment of freight specified therein apply to and govern the rights of the providing that loss/damage caused by negligence of charterers or
parties concerned in this shipment.xxx repairers are excluded from coverage. William Lines brought the
vessel to Cebu Shipyard for annual dry-docking and repair. The two
The provision on arbitration in the Charter Party reads:
executed contracts stipulating the liabilities of both parties, including
4. Arbitration. Any dispute arising from the making, performance or that the insurance on the vessel should be maintained by the owner
termination of this Charter Party shall be settled in New York, Owner and during period of the contract. After the vessel was transferred to the
Charterer each appointing an arbitrator, who shall be a merchant, broker docking quay, it caught fire and sank, resulting to its eventual total
loss. Cebu Shipyard claims that it is a co-assured under the Marine
or individual experienced in the shipping business; the two thus chosen,
Hull Insurance Policy by virtue of Clause 20, and therefore no
if they cannot agree, shall nominate a third arbitrator who shall be an
subrogation can be made by Prudential. SC held that it is not and the
admiralty lawyer. Such arbitration shall be conducted in conformity with
petition was denied.
the provisions and procedure of the United States arbitration act, and a DOCTRINE: Intention of parties to make each other co-assured is to
judgment of the court shall be entered upon any award made by said be gleaned from the insurance policy itself and not from any other
arbitrator. Nothing in this clause shall be deemed to waive Owner's right contract because the policy denominates the assured and the
to lien on the cargo for freight, deed of freight, or demurrage. beneficiaries.
FACTS:
Clearly, the Bill of Lading incorporates by reference the terms of the Charter
Party. It is settled law that the charter may be made part of the contract under  Cebu Shipyard and Engineering Works, Inc. (CSEW) is engaged in the
which the goods are carried by an appropriate reference in the Bill of Lading. As business of dry-docking and repairing of marine vessels while the

Page 11 of 47
Prudential Guarantee and Assurance, Inc. (Prudential) is in the non-life  CSEW claims that the insurance policy does not cover loss resulting from
insurance business. the fault of negligent charterers that are assured in the same policy and
 William Lines, Inc. is in the shipping business. It was the owner of M/V by virtue of clause 20, it is deemed a co-assured.
Manila City, a luxury passenger-cargo vessel, which caught fire and sank.
 At the time of the unfortunate occurrence sued upon, subject vessel was ISSUE/S & RATIO:
insured with Prudential for P45M for hull and machinery. The Hull Policy
included an “Additional Perils (INCHMAREE)” Clause covering loss of or 1. WON CSEW is co-assured, thus losses caused by it are not covered
damage to the vessel through the negligence of, among others, ship by the policy- NO
repairmen a. The fact that clause 20 benefited petitioner, does not automatically
 Petitioner CSEW was also insured by Prudential for third party liability make it a co-assured of William Lines.
under a Shiprepairer’s Legal Liability Insurance Policy. The policy was for b. Intention of parties to make each other co-assured is to be gleaned
P10 million only, under the limited liability clause from the insurance policy itself and not from any other contract
 On Feb. 5, 1991, William Lines, Inc. brought its vessel, M/V Manila City, because the policy denominates the assured and the beneficiaries.
to the Cebu Shipyard in Lapulapu City for annual dry-docking and repair. c. Prudential named only William Lines, Inc. as the assured. There
 On Feb. 6, 1991, an arrival conference was held between representatives was no manifestation of any intention of William Lines Inc to make
of William Lines, Inc. and CSEW to discuss the work to be undertaken on CSEW a co-assured. When the terms of a contract are clear, its
the M/V Manila City. The contracts, denominated as Work Orders, were stipulations control.
signed thereafter., with the following stipulations: d. If CSEW were deemed co-assured, it would nullify any claim of
o 10. The Contractor shall replace at its own work and at its own William Lines Inc. No shipowner would agree to make shiprepairer
cost any work or material which can be shown to be defective and a co-assured because any claim it has under the policy would be
which is communicated in writing invalidated. Such result could not have been intended by William
o 20. The insurance on the vessel should be maintained by the Lines Inc.
customer and/or owner of the vessel during the period the contract 2. WON CSEW had “management and supervisory control“ of the ship at the
is in effect. time the fire broke out- YES
o The total liability of the Contractor to the Customer or of any sub- a. The factual findings by the CA are conclusive on the parties and
contractor shall be limited in respect of any defect or event to the are not reviewable by this Court.
sum of 1M. 3. WON the doctrine of res ipsa loquitur applies against the crew- YES
 While the M/V Manila City was undergoing dry-docking and repairs within a. For the doctrine of res ipsa loquitur to apply to a given situation,
the premises of CSEW, the master, officers and crew of M/V Manila City the following conditions must concur: (1) the accident was of a
stayed in the vessel, using their cabins as living quarters. Other kind which does not ordinarily occur unless someone is negligent;
employees hired by William Lines to do repairs and maintenance work on and (2) that the instrumentality or agency which caused the injury
the vessel were also present during the dry-docking. was under the exclusive control of the person charged with
 On February 16, 1991, after subject vessel was transferred to the docking negligence.
quay, it caught fire and sank, resulting to its eventual total loss b. The facts and evidence reveal the presence of these conditions.
 On February 21, 1991, William Lines, Inc. filed a complaint for damages First, the fire would not have happened in the ordinary course of
against CSEW, alleging that the fire which broke out in M/V Manila City things if reasonable care and diligence had been exercised.
was caused by CSEWs negligence and lack of care. 4. WON the provisions limiting CSEW’s liability for negligence to a maximum
 Prudential was impleaded as co-plaintiff, after it paid William Lines, Inc. of Php 1 million are valid- NO
the value of the hull and machinery insurance on the M/V Manila City. As a. Although contracts of adhesion have been consistently upheld as
a result of such payment Prudential was subrogated to the claim of P45 valid, reliance on such contracts cannot be favored especially
million, representing the value of the said insurance it paid. where the facts and circumstances warrant that subject
 Trial Court: CSEW to pay William Lines and Prudential (45M) stipulations be disregarded. Tthe facts and circumstances vis-a-
 CA: Affirmed TC. Ordered the partial dismissal of the case insofar as vis the nature of the provision sought to be enforced should be
CSEW and William Lines were concerned. considered, bearing in mind the principles of equity and fair play.

Page 12 of 47
RULING: Petition denied. the intent and purpose of the parties. CA was correct in holding petitioner should
reimburse respondent ₱ 5,000.00.
MANILA MAHOGANY V CA
When Manila Mahogany executed another release claim discharging SMC from
FACTS: all rights of action after the insurer had paid the proceeds of the policy – the
compromise agreement of ₱ 5,000.00– the insurer is entitled to recover from the
Petitioner insured its Mercedes Benz 4-door sedan with respondent insurance insured the amount of insurance money paid. Petitioner by its own acts released
company . The insured vehicle was bumped and damaged by a truck owned by SMC, thereby defeating respondent’s right of subrogation, the right of action
San Miguel Corporation (SMC). For the damage caused, respondent company against the insurer was also nullified.
paid petitioner ₱ 5,000.00 in amicable settlement. Petitioner’s general manager
executed a Release of Claim, subrogating respondent company to all its right to Since the insurer can be subrogated to only such rights as the insured may have,
action against San Miguel Corp. Respondent company wrote the Insurer should the insured, after receiving payment from the insurer, release the
Adjusters, Inc. to demand reimbursements from San Miguel Corporation of the wrongdoer who caused the loss, the insurer losses his rights against the latter.
amount it had paid petitioner. Insurer Adjusters, Inc. refused reimbursement But in such a case, the insurer will be entitled to recover from the insured
alleging that SMC had already paid petitioner ₱ 4,500.00 for the damages to whatever it has paid to the latter, unless the release was made with the consent
petitioner’s motor vehicle, as evidenced by a cash voucher and Release of Claim of the insurer.
executed by the General Manager of petitioner discharging SMC from “ all
actions, claims, demands the right of action that now exist or hereafter develop
FF CRUZ AND CO vs. CA
arising out of or as a consequence of the accident.

Respondent demanded the ₱ 4,500.00 amount from petitioner. Petitioner Facts:


refused. Suit was filed for recovery. City Court ordered petitioner to pay A fire broke up from the furniture shop of the petitioner in Caloocan city early
respondent. CFI affirmed. CA affirmed with modification that petitioner was to September 6, 1974. Prior to that, neighbor of the said shop requested that the
pay respondent the total amount of ₱ 5,000.00 it had received from respondent. petitioner should build a firewall but failed to do so. The cause of the fire was
never discovered. Private respondent got P35k from the insurance on their
house and contents thereof.
Petitioner’s argument: Since the total damages were valued at P9,486.43 and
only ₱ 5,000.00 was received by petitioner from respondent, petitioner argues
Issue:
that it was entitled to go after SMC to claim the additional which was eventually
Whether or not the 35k be deducted from the damages thereof
paid to it.
Ruling:
Respondent’s argument: No qualification to its right of subrogation. Since P35k had already been claimed by the respondents, the court held that
such amount should be deducted from the award of damages in accordance
ISSUE: with Art 2207 NCC
Art. 2207. If the plaintiff’s property has been insured, and he has received
Whether or not the insured should pay the insurer despite that the subrogation indemnity from the insurance company for the injury or loss arising out of the
in the Release of Claim was conditioned on recovery of the total amount of wrong or breach of contract complained of, the insurance company shall be
damages that the insured has sustained. subrogated to the rights of the insured against the wrongdoer or the person
who has violated the contract. If the amount paid by the insurance company
does not fully cover the injury or loss, the aggrieved party shall be entitled to
RULING:
recover the deficiency from the person causing the loss or injury.
Having been indemnified by their insurer, private respondents are entitled only
NO. Supreme Court said there being no other evidence to support its allegation to recover the deficiency from the petitioner.
that a gentleman’s agreement existed between the parties, not embodied in the Whether or not the insurer should exercise the rights of the insured to which it
Release of Claim, such Release of Claim must be taken as the best evidence of had been subrogated lies solely within the former’s sound discretion. Since the
Page 13 of 47
insurer is not a party to the case, its identity is not of record and no claim is HELD: 1.Yes. While the participation of the insured in the boxing contest is
made on its behalf, the private respondent’s insurer has to claim his right to voluntary, the injury was sustained when he slid, giving occasion to the
reimbursement of the P35,000.00 paid to the insured. infliction by his opponent of the blow that threw him to the ropes of the ring.
RATIO: 1. The terms "accident" and "accidental", as used in insurance
contracts, have not acquired any technical meaning, and are construed by the
courts in their ordinary and common acceptation. Thus, the terms have been
WEEK 3 taken to mean that which happen by chance or fortuitously, without intention
and design, and which is unexpected, unusual, and unforeseen. An accident is
Dela Cruz vs. Capital Insurance & Surety Co. an event that takes place without one's foresight or expectation

FACTS: 1. an event that proceeds from an unknown cause, or is an unusual effect of a
Eduardo de la Cruz, employed as a mucker in the Itogon-Suyoc Mines, Inc. in known cause and, therefore, not expected.
Baguio, was the holder of an accident insurance policy (No. ITO-BFE-170) 2. The generally accepted rule is that, death or injury does not result from
underwritten by the Capital Insurance & Surety Co., Inc., for the period accident or accidental means within the terms of an accident-policy if it is the
beginning November 13, 1956 to November 12, 1957. natural result of the insured's voluntary act, unaccompanied by anything
2. On January 1, 1957, the Itogon-Suyoc Mines, Inc. sponsored a boxing unforeseen except the death or injury.
contest for general entertainment. 3. Death or disablement resulting from engagement in boxing contests was
3. Insured Eduardo de la Cruz, a non-professional boxer participated. not declared outside of the protection of the insurance contract. Failure of the
4.In the course of his bout with another person, likewise a non-professional, of defendant insurance company to include death resulting from a boxing match
the same height, weight, and size, Eduardo slipped and was hit by his or other sports among the prohibitive risks leads inevitably to the conclusion
opponent on the left part of the back of the head, causing Eduardo to fall, with that it did not intend to limit or exempt itself from liability for such death.
his head hitting the rope of the ring. DOCTRINE
5. He was brought to the Baguio General Hospital the following day. The cause In other words, where the death or injury is not the natural or probable result
of death was reported as hemorrhage, intracranial, left. of the insured's voluntary act, or if something unforeseen occurs in the doing of
6. Simon de la Cruz, the father of the insured and who was named beneficiary the act which produces the injury, the resulting death is within the protection
under the policy, filed a claim with the insurance company for payment of the of policies insuring against death or injury from accident.
indemnity under the insurance policy.
7. As the claim was denied, De la Cruz instituted the action in the Court of First Sun v CA G.R. No. 92383 July 17, 1992
Instance of Pangasinan for specific performance. J. Cruz
8. Defendant insurer set up the defense that the death of the insured, caused Facts:
by his participation in a boxing contest, was not accidental and, therefore, not Lim accidentally killed himself with his gun after removing the magazine,
covered by insurance. showing off, pointing the gun at his secretary, and pointing the gun at his
9. After due hearing the court rendered the decision in favor of the plaintiff temple. The widow, the beneficiary, sued the petitioner and won 200,000 as
which is the subject of the present appeal. indemnity with additional amounts for other damages and attorney’s fees. This
10. Eduardo was insured "against death or disability caused by accidental was sustained in the Court of Appeals then sent to the Supreme court by the
means". insurance company.
11. Appellant insurer now contends that while the death of the insured was due Issue:
to head injury, said injury was sustained because of his voluntary participation 1. Was Lim’s widow eligible to receive the benefits?
in the contest. 2. Were the other damages valid?
12. It is claimed that the participation in the boxing contest was the "means" Held:
that produced the injury which, in turn, caused the death of the insured. And, 1. Yes 2. No
since his inclusion in the boxing card was voluntary on the part of the insured, Ratio: 1. There was an accident.
he cannot be considered to have met his death by "accidental means". De la Cruz v. Capital Insurance says that "there is no accident when a
ISSUE: 1. WON the death of Eduardo de la Cruz is covered by accident deliberate act is performed unless some additional, unexpected, independent
insurance policy.
Page 14 of 47
and unforeseen happening occurs which produces or brings about their injury Held.
or death." This was true when he fired the gun. NO.
Under the insurance contract, the company wasn’t liable for bodily injury SC already ruled in the case of Ty v. FNSI that were the insurance policies
caused by attempted suicide or by one needlessly exposing himself to danger define partial disability as loss of either hand by amputation through the bones
except to save another’s life. of the wrist, the insured cannot recover under said policies for temporary
Lim wasn’t thought to needlessly expose himself to danger due to the witness disability of his left hand caused by the fractures of some fingers. The
testimony that he took steps to ensure that the gun wasn’t loaded. He even provision is clear enough to inform the party entering into that contract that
assured his secretary that the gun was loaded. the loss to be considered a disability entitled to indemnity, must be severance
There is nothing in the policy that relieves the insurer of the responsibility to or amputation of the affected member of the body of the insured.
pay the indemnity agreed upon if the insured is shown to have contributed to
his own accident. Calanoc v. CAG.R. No. L-8151 December 16, 1955J.
2. “In order that a person may be made liable to the payment of moral
damages, the law requires that his act be wrongful. The adverse result of an Bautista Angelo
action does not per se make the act wrongful and subject the act or to the Doctrine: In case of ambiguity in an insurance contract covering accidental
payment of moral damages. The law could not have meant to impose a penalty death, the Supreme Court held that such terms shall be construed strictly
on the right to litigate; such right is so precious that moral damages may not against the insurer and liberally in favor of the insured in order to effect the
be charged on those who may exercise it erroneously. For these the law taxes purpose of indemnity.
costs.” Facts: Melencio Basilio, a watchman of the Manila Auto Supply, secured a life
If a party wins, he cannot, as a rule, recover attorney's fees and litigation insurance policy from the Philippine American Insurance Company in the
expenses, since it is not the fact of winning alone that entitles him to recover amount of P2,000 to which was attached a supplemental contract covering
such damages of the exceptional circumstances enumerated in Art. 2208. death by accident. He later died from a gunshot wound on the occasion of a
Otherwise, every time a defendant wins, automatically the plaintiff must pay robbery committed; subsequently, his widow was paid P2,000 representing the
attorney's fees thereby putting a premium on the right to litigate which should face value of the policy. The widow demanded the payment of the additional
not be so. For those expenses, the law deems the award of costs as sufficient.” sum of P2,000 representing the value of the supplemental policy which the
company refused because the deceased died by murder during the robbery and
Ty v. Filipinas Compañia de Seguros - Insurance Policy while making an arrest as an officer of the law which were expressly excluded
17 SCRA 364 in the contract. The company’s contention which was upheld by the Court of
Facts: Appeals provides that
> Ty was employed as a mechanic operator by Braodway Cotton Factory at The circumstances surrounding Basilio’s death was caused by one of the risks
Grace Park, Caloocan. excluded by the supplementary contract which exempts the company from
> In 1953, he took personal accident policies from 7 insurance companies (6 liability.
defendants), on different dates, effective for 12 mos.
> On Dec. 24. 1953, a fire broke out in the factory were Ty was working. A Issue: Is the Philippine American Life Insurance Co. liable to the petitioner for
hevy object fell on his hand when he was trying to put out the fire. the amount covered by the supplemental contract?
> From Dec. 1953 to Feb. 6, 1954 Ty received treatment at the Nat’l
Orthopedic Hospital for six listed injuries. The attending surgeon certified that Held: Yes.
these injuries would cause the temporary total disability of Ty’s left hand. The circumstances of Basilio’s death cannot be taken as purely intentional on
> Insurance companies refused to pay Ty’s claim for compensation under the the part of Basilio to expose himself to the danger. There is no proof that his
policies by reason of said disability of his left hand. Ty filed a complaint in the death was the result of intentional killing because there is the possibility that
municipal court who decided in his favor. the malefactor had fired the shot merely to scare away the
> CFI reversed on the ground that under the uniform terms of the policies, people around. In this case, the company’s defense points out that Basilio’s is
partial disability due to loss of either hand of the insured, to be compensable included among the risks excluded in the supplementary contract; however,
must be the result of amputation. the terms and phraseology of the exception clause should be clearly expressed
Issue: within the understanding of the insured. Art. 1377 of the New Civil Code
Whether or not Ty should be indemnified under his accident policies. provides that in case ambiguity, uncertainty or obscurity in the interpretation
Page 15 of 47
of the terms of the contract, it shall be construed against the party who caused Master Policy No. 2005 and Individual Policy No. 08924 with his parents,
such obscurity. Applying this to the situation, the ambiguous or obscure terms spouses
in the insurance policy are to be construed strictly against the insurer and Julia and Carlos Surposa and brothers Christopher, Charles, Chester and
liberally in favor of the insured party. The reason is to ensure the protection of Clifton, all surnamed Surposa, as beneficiaries.
the insured since these insurance contracts are usually arranged and employed While said insurance policy was in full force and effect, the insured Carlie
by experts and legal advisers acting exclusively in the interest of the insurance Surposa, died on October 18, 1988 as a result of a stab wound inflicted by one
company. As long as insurance companies insist upon the use of ambiguous, of the three (3) unidentified men without provocation and warning on the part
intricate and technical provisions, which conceal their own intentions, the of the former as... he and his cousin, Winston Surposa, were waiting for a ride
courts must, in fairness to those who purchase insurance, construe every on their way home along Rizal-Locsin Streets, Bacolod City after attending the
ambiguity in favor of the insured. celebration of the "Maskarra Annual Festival."... private respondent and the
other beneficiaries of said insurance policy filed a written notice of claim with
the petitioner insurance company which denied said claim contending that
Biagtan vs. The Insular Life Assurance Company, Ltd. (winner) 44 SCRA murder and assault are not within the scope of the coverage of the insurance...
58 policy.
Facts: Insurance Commission ruled in favor of insured/beneficiaries
Juan S. Biagtan was insured with defendant Insular Life Assurance Company On February 24, 1989, private respondent filed a complaint with the Insurance
under Policy No. 398075 for the sum of P5,000.00 and, under a supplementary Commission
contract denominated "Accidental Death Benefit Clause, for an additional sum "In the light of the foregoing, we find respondent liable to pay complainant the
of P5,000.00 if "the death of the Insured resulted directly from bodily injury sum of P15,000.00 representing the proceeds of the policy with interest. As no
effected solely through external and violent means sustained in an accident . . . evidence was submitted to prove the claim for mortuary aid in the sum of
and independently of all other causes." The clause, however, expressly P1,000.00, the same... cannot be entertained.
provided that it would not apply where death resulted from an injury On July 11, 1991, the appellate court affirmed said decision.
"intentionally inflicted by a third party."library petitioner filed this petition alleging grave abuse of discretion on the part of the
On the night of May 20, 1964 or during the first hours of the following day a appellate court in applying the principle of "expresso unius exclusion alterius"
band of robbers entered the house of the insured Juan S. Biagtan. in a personal accident insurance policy... since death resulting from murder
Issue: and/or assault are impliedly excluded in said insurance policy considering that
Whether the wounds received by the insured at the hands of the robbers were the cause of death of the insured was not accidental but rather a deliberate and
inflicted intentionally. intentional act of the assailant in killing the former as indicated by the
Held: location... of the lone stab wound on the insured.
Yes. But where a gang of robbers enter a house and coming face to face with Therefore, said death was committed with deliberate intent which, by the very
the owner, even if unexpectedly, stab him repeatedly, it is contrary to all nature of a personal accident insurance policy, cannot be indemnified.
reason and logic to say that his injuries are not intentionally inflicted, Issues:
regardless of whether they prove fatal or not. As it was, in the present case WON the death of the insured was committed...... with deliberate intent which,
they did prove fatal, and the robbers have been accused and convicted of the by the very nature of a personal accident insurance policy, cannot be
crime of robbery with homicide. Under the circumstance, the insurance indemnified
company was correct in refusing to pay the additional sum of P2,000.00 under Ruling:
the accidental death benefit clause which expressly provided that it would not We do not agree.
apply where death resulted from an injury "intentionally" inflicted by a third In the case at bar, it cannot, be pretended that Carlie Surposa died in the
party. course of an assault or murder as a result of his voluntary act considering the
very nature of these crimes.
FINMAN GENERAL ASSURANCE CORPORATION v. CA, GR No. 100970, the personal accident insurance policy, involved herein specifically enumerated
1992-09-02 only ten (10) circumstances wherein no liability attaches to petitioner insurance
Facts: company for any injury, disability or loss suffered by the insured as a result of
October 22, 1986, deceased Carlie Surposa was insured with petitioner Finman any of the... stipulated causes. The principle of "expresso unius exclusio
General Assurance Corporation under Finman General Teachers Protection Plan alterius" -- the mention of one thing implies the exclusion of another thing -- is
Page 16 of 47
therefore applicable in the instant case since murder and assault, not having (1) Whether or not the policies should be avoided for the reason that there was
been expressly included in the enumeration of the... circumstances that would a breach of warranty.
negate liability in said insurance policy cannot be considered by implication to
discharge the petitioner insurance company from liability for any injury, Under the Memorandum of Warranty, there should be no less than 1 hydrant
disability Or loss suffered by the insured. for each 150 feet of external wall measurements of the compound, and since
Thus, the failure of the petitioner insurance company... to include death bodegas insured had an external wall per meter of 1640 feet, the insured
resulting from murder or assault among the prohibited risks leads inevitably to should have 11 hydrants in the compound. But he only had 2.
the conclusion that it did not intend to limit or exempt itself from liability for
such death. Even so, the insurer is barred by estoppel to claim violation of the fire hydrants
Principles: warranty, because knowing that the number of hydrants it demanded never
The terms 'accident' and 'accidental', as used in insurance contracts have not existed from the very beginning, appellant nevertheless issued the policies
acquired any technical meaning, and are construed by the courts in their subject to such warranty and received the corresponding premiums. The
ordinary and common acceptation. Thus, the terms have been taken to mean insurance company was aware, even before the policies were issued, that in
that which happen by chance or... fortuitously, without intention and design, the premises there were only 2 hydrants and 2 others were owned by the
and which is unexpected, unusual, and unforeseen. An accident is an event Municipality, contrary to the requirements of the warranties in question.
that takes place without one's foresight or expectation -- an event that
proceeds from an unknown cause, or is an unusual effect of a known cause It should be close to conniving at fraud upon the insured to allow the insurer to
and, therefore,... not expected." claim now as void the policies it issued to the insured, without warning him of
The generally accepted rule is that, death or injury does not result from the fatal defect, of which the insurer was informed, and after it had misled the
accident or accidental means within the terms of an accident-policy if it is the insured into believing that the policies were effective.
natural result of the insured's voluntary act, unaccompanied by anything
unforeseen except the death or... injury. Accdg to American Jurisprudence: It is a well-settled rule that the insurer at
the time of the issuance of a policy has the knowledge of existing facts, which
Qua Chee Gan v. Law Union Rock - Breach of Warranty if insisted on, would invalidate the contract from its very inception, such
98 PHIL 85 knowledge constitutes a waiver of conditions in the contract inconsistent with
Facts: known facts, and the insurer is stopped thereafter from asserting the breach of
> Qua Chee Gan, a merchant, owned 4 warehouses in Albay which were used such conditions. The reason for the rule is: To allow a company to accept one’s
for the storage or copra and hemp in which the appelle deals with exclusively. money for a policy of insurance which it knows to be void and of no effect,
> The warehouses together with the contents were insured with Law Union though it knows as it must that the insured believes it to be valid and binding is
since 1937 and the loss made payable to PNB as mortgagee of the hemp and so contrary to the dictates of honesty and fair dealing, as so closely related to
copra. positive fraud, as to be abhorrent to fair-minded men. It would be to allow the
> A fire of undetermined cause broke out in July 21, 1940 and lasted for company to treat the policy as valid long enough to get the premium on it, and
almost 1 whole week. leave it at liberty to repudiate it the next moment.
> Bodegas 1, 3, and 4 including the merchandise stored were destroyed
completely. Moreover, taking into account the well-known rule that ambiguities or
> Insured then informed insurer of the unfortunate event and submitted the obscurities must strictly be interpreted against the party that cause them, the
corresponding fire claims, which were later reduced to P370T. memorandum of warranty invoked by the insurer bars the latter from
> Insurer refused to pay claiming violations of the warranties and conditions, questioning the existence of the appliances called for, since its initial
filing of fraudulent claims and that the fire had been deliberately caused by the expression “the undernoted appliances for the extinction of fire being kept on
insured. the premises insured hereby..” admits of the interpretation as an admission of
> Insured filed an action before CFI which rendered a decision in favor of the the existence of such appliances which insurer cannot now contradict, should
insured. the parole evidence apply.

Issues and Resolutions:

Page 17 of 47
(2) Whether or not the insured violated the hemp warranty provision against Lessons Applicable: Ambiguous Provisions Interpreted Against Insurer
the storage of gasoline since insured admitted there were 36 cans of gasoline (Insurance)
in Bodega 2 which was a separate structure and not affected by the fire. FACTS:
April 13, 1957: Simeon del Rosario, father of the insured who died from
It is well to note that gasoline is not specifically mentioned among the drowning filed a claim for payment with Equitable Ins. and Casualty Co., Inc.
prohibited articles listed in the so-called hemp warranty. The clause relied but it refused to pay more than P1,000 php so a case was filed with the RTC for
upon by the insurer speaks of “oils”. Ordinarily, oils mean lubricants and not the P2,000 balance stating that under the policy they are entitled to P1,000 to
gasoline or kerosene. Here again, by reason of the exclusive control of the P3,000 as indemnity
insurance company over the terms of the contract, the ambiguity must be held RTC: entitled to recover P3,000 - policy does not positively state any definite
strictly against the insurer and liberally in favor of the insured, specially to amount, there is an ambiguity in this respect in the policy, which ambiguity
avoid a forfeiture. must be interpreted in favor of the insured and strictly against the insurer so as
to allow greater indemnity
Furthermore, the gasoline kept was only incidental to the insured’s business. It ISSUE: W/N Simeon is entitled to recover P3,000
is a well settled rule that keeping of inflammable oils in the premises though HELD: YES.
prohibited by the policy does NOT void it if such keeping is incidental to the terms in an insurance policy, which are ambiguous, equivocal or uncertain are
business. Also, the hemp warranty forbade the storage only in the building to to be construed strictly against, the insurer, and liberally in favor of the insured
which the insurance applies, and/or in any building communicating therewith; so as to effect the dominant purpose of indemnity or payment to the insured,
and it is undisputed that no gasoline was stored in the burnt bodegas and that especially where a forfeiture is involved
Bodega No. 2 which was where the gasoline was found stood isolated from the reason for this rule is that the "insured usually has no voice in the selection or
other bodegas. arrangement of the words employed and that the language of the contract is
selected with great care and deliberation by expert and legal advisers
Del Rosario v. Equitable Insurance - Life Insurance Policy employed by, and acting exclusively in the interest of, the insurance company
118 PHIL 349
Facts: Verendia v. CA - Insurance Policy
> Equitable Insurance issued a life Insurance policy to del Rosario binding 217 SCRA 1993
itself to pay P1,000 to P3,000 as indemnity. Facts:
> Del Rosario died in a boating accident. The heirs filed a claim and Equitable > Fidelity and Surety Insurance Company (Fidelity) issued Fire Insurance
paid them P1,000. Policy No. F-18876 effective between June 23, 1980 and June 23, 1981
> The heir filed a complaint for recovery of the balance of P2,000, claiming covering Rafael (Rex) Verendia's residential in the amount of P385,000.00.
that the insurere should pay him P3,000 as stated in the policy. Designated as beneficiary was the Monte de Piedad & Savings Bank.
Issue: > Verendia also insured the same building with two other companies, namely,
Whether or not the heir is entitled to recover P3,000. The Country Bankers Insurance for P56,000.00 and The Development
Held: Insurance for P400,000.00.
YES. > While the three fire insurance policies were in force, the insured property
Generally accepted principles or ruling on insurance, enunciate that where was completely destroyed by fire.
there is an ambiguity with respect to the terms and conditions of the policy, the > Fidelity appraised the damage amounting to 385,000 when it was
same shall be resolved against the one responsible thereof. The insured has accordingly informed of the loss. Despite demands, Fidelity refused payment
little, if any, participation in the preparation of the policy. The interpretation of under its policy, thus prompting Verendia to file a complaint for the recovery of
obscure stipulations in a contract should not favor the party who cause the 385,000
obscurity. > Fidelity, averred that the policy was avoided by reason of over-insurance,
that Verendia maliciously represented that the building at the time of the fire
Insurance Case Digest: Del Rosario v. Equitable Ins. and Casualty Co., was leased under a contract executed on June 25, 1980 to a certain Roberto
Inc. (1963) Garcia, when actually it was a Marcelo Garcia who was the lessee.
G.R. No. L-16215 June 29, 1963
Issue:
Page 18 of 47
Whether or not Verendia can claim on the insurance despite the There is also no reason to conclude that by submitting the subrogation receipt
misrepresentation as to the lessee and the overinsurance. as evidence in court, Fidelity bound itself to a "mutual agreement" to settle
Verendia's claims in consideration of the amount of P142,685.77. While the
Held: said receipt appears to have been a filled-up form of Fidelity, no representative
NOPE. of Fidelity had signed it. It is even incomplete as the blank spaces for a witness
The contract of lease upon which Verendia relies to support his claim for and his address are not filled up. More significantly, the same receipt states
insurance benefits, was entered into between him and one Robert Garcia, a that Verendia had received the aforesaid amount. However, that Verendia had
couple of days after the effectivity of the insurance policy. When the rented not received the amount stated therein, is proven by the fact that Verendia
residential building was razed to the ground, it appears that Robert Garcia was himself filed the complaint for the full amount of P385,000.00 stated in the
still within the premises. However, according to the investigation by the police, policy. It might be that there had been efforts to settle Verendia's claims, but
the building appeared to have "no occupants" and that Mr. Roberto Garcia was surely, the subrogation receipt by itself does not prove that a settlement had
"renting on the otherside of said compound" These pieces of evidence belie been arrived at and enforced. Thus, to interpret Fidelity's presentation of the
Verendia's uncorroborated testimony that Marcelo Garcia whom he considered subrogation receipt in evidence as indicative of its accession to its "terms" is
as the real lessee, was occupying the building when it was burned. not only wanting in rational basis but would be substituting the will of the Court
Ironically, during the trial, Verendia admitted that it was not Robert Garcia who for that of the parties
signed the lease contract but it was Marcelo Garcia cousin of Robert, who had
also been paying the rentals all the while. Verendia, however, failed to explain
why Marcelo had to sign his cousin's name when he in fact he was paying for
the rent and why he (Verendia) himself, the lessor, allowed such a ruse.
Fidelity's conclusions on these proven facts appear, therefore, to have sufficient Fortune Insurance And Surety Co., Inc. V. CA
bases: Verendia concocted the lease contract to deflect responsibility for the
fire towards an alleged "lessee", inflated the value of the property by the
Lessons Applicable: Stipulations Cannot Be Segregated (Insurance)
alleged monthly rental of P6,500) when in fact, the Provincial Assessor of Rizal
had assessed the property's fair market value to be only P40,300.00, insured
the same property with two other insurance companies for a total coverage of FACTS:
around P900,000, and created a dead-end for the adjuster by the
 Producers Bank of the Philippines insured with Fortune Insurance and Surety
disappearance of Robert Garcia.
Basically a contract of indemnity, an insurance contract is the law between the Co. P725,000 which was lost during a robbery of Producer's armored
parties. Its terms and conditions constitute the measure of the insurer's liability vehicle while it was in transit from Pasay City City to its Makati head office.
and compliance therewith is a condition precedent to the insured's right to
recovery from the. As it is also a contract of adhesion, an insurance contract  The armored car was driven by Benjamin Magalong Y de Vera, escorted by
should be liberally construed in favor of the insured and strictly against the Security Guard Saturnino Atiga Y Rosete.
insurer company which usually prepares it.  After an investigation conducted by the Pasay police authorities, the driver
Considering, however, the foregoing discussion pointing to the fact that
Verendia used a false lease contract to support his claim under Fire Insurance Magalong and guard Atiga were charged, together with Edelmer Bantigue Y
Policy, the terms of the policy should be strictly construed against the insured. Eulalio, Reynaldo Aquino and John Doe, with violation of P.D. 532 (Anti-
Verendia failed to live by the terms of the policy, specifically Section 13 thereof
Highway Robbery Law)
which is expressed in terms that are clear and unambiguous, that all benefits
under the policy shall be forfeited "if the claim be in any respect fraudulent, or  Upon claiming, Fortune refused stating that it is not liable since under the
if any false declaration be made or used in support thereof, or if any fraudulent general exceptions of the policy:
means or devises are used by the Insured or anyone acting in his behalf to
 any loss caused by any dishonest, fraudulent or criminal act of the insured
obtain any benefit under the policy". Verendia, having presented a false
declaration to support his claim for benefits in the form of a fraudulent lease or any officer, employee, partner, director, trustee or authorized
contract, he forfeited all benefits therein by virtue of Section 13 of the policy in
the absence of proof that Fidelity waived such provision
Page 19 of 47
representative of the Insured whether acting alone or in conjunction with  Misamis Lumber Corporation (Misamis), formerly Lanao Timber Mills, Inc.,
others. . . . insured its Ford Falcon motor car with Capital Insurance & Surety Company
 RTC: favored Producers Bank since Driver and Security Guard were merely (Capital)
assigned  November 25, 1961 11 pm: The car broke when it hit a hollow block lying
 CA: Affirmed RTC alongside the water hole which the driver did not see because the on-coming
ISSUE: W/N the driver and security guard are employees under the general car did not dim its light
exception  The car was towed and repaired by Morosi Motors costing P302.27
 November 29, 1961: After the repairs were made, Misamis made a report to
HELD: YES. Petition is granted. Capital who only admits liability of P150
 It is clear to us that insofar as Fortune is concerned, it was its intention to  CFI: paragraph 4 of the policy is clear and specific and leaves no room for
exclude and exempt from protection and coverage losses arising from interpretation that the repair liability is limited to P150
dishonest, fraudulent, or criminal acts of persons granted or having ISSUE: W/N Misamis is entitled to an amount exceeding P150
unrestricted access to Producers' money or payroll. When it used then the
term "employee," it must have had in mind any person who qualifies as such HELD: NO.
as generally and universally understood, or jurisprudentially established in  insurance contract may be rather onerous (one-sided) but that in itself does
the light of the four standards in the determination of the employer-employee not justify the abrogation of its express terms, terms which the insured
relationship, 21 or as statutorily declared even in a limited sense as in the accepted or adhered to and which is the law between the contracting parties
case of Article 106 of the Labor Code which considers the employees under Misamis Lumber vs. Capital Insurance, G.R. L-21380, May 20, 1966
a "labor-only" contract as employees of the party employing them and not of
the party who supplied them to the employer Facts:
 Producers entrusted the three with the specific duty to safely transfer the
money to its head office, with Alampay to be responsible for its custody in The vehicle owned by the petitioner was covered by an insurance policy issued by the
transit; Magalong to drive the armored vehicle which would carry the money; respondent. In such policy it contained instructions and details on how to proceed with
and Atiga to provide the needed security for the money, the vehicle, and his the claim for repairs.
two other companions.
 A "representative" is defined as one who represents or stands in the place of When the time came for the vehicle to be repaired, the petitioner took it upon himself to
another; one who represents others or another in a special capacity, as an have the vehicle repaired without the authority from the insurer, in the policy if such an
agent, and is interchangeable with "agent." event is to occur, and a repair was done without the authority of the insurer, its liability
is limited only to 150 pesos. The repair bill exceeded such amount.
Misamis Lumber Corp. V. Capital Ins. And Surety Co.,

Upon filing of claim the insurer refused payment in excess of 150 pesos.
Lessons Applicable: Judicial Construction Cannot Alter Terms (Insurance)

Issue(s):
FACTS:
Page 20 of 47
The Court of Appeals affirmed the decision of the Insurance Commissioner. In
its decision, the appellate court distinguished between P & I Clubs vis-à-vis
1.) Can the respondent be made to reimburse the petitioner for the actual cost of repairs
conventional insurance. The appellate court also held that Pioneer merely acted
which exceed the repair limit amount? as a collection agent of Steamship Mutual.

Hence this petition by White Gold.


Ruling:
Issues:

The lower court’s recourse to legal hermeneutics is not called for because paragraph 4 of 1. Is Steamship Mutual, a P & I Club, engaged in the insurance business in the
the policy is clear and specific and leaves no room for interpretation. The interpretation Philippines?

is even unjustified because it opposes what was specifically stipulated. Thus it will be 2. Does Pioneer need a license as an insurance agent/broker for Steamship
observed that the policy drew out not only the limits of the insurer’s liability but also the Mutual?
mechanics that the insured had to follow to be entitled to full indemnity for repairs. The Held: Yes. Petition granted.
option to undertake repairs is accorded to the insurance company per paragraph 2. The
Ratio:
said company was deprived of the option because the insured took it upon itself to have
the repairs made, and only notified the insurer when the repairs are done. As a White Gold insists that Steamship Mutual as a P & I Club is engaged in the
insurance business. To buttress its assertion, it cites the definition as “an
consequence, paragraph 4, which limits the company’s liability to P150.00 appliesWEEK association composed of shipowners in general who band together for
4 the specific purpose of providing insurance cover on a mutual basis against
liabilities incidental to shipowning that the members incur in favor of third
White Gold v Pioneer G.R. No. 154514. July 28, 2005 parties.”

Facts: They argued that Steamship Mutual’s primary purpose is to solicit and provide
protection and indemnity coverage and for this purpose, it has engaged the
White Gold procured a protection and indemnity coverage for its vessels from services of Pioneer to act as its agent.
The Steamship Mutual through Pioneer Insurance and Surety
Corporation. White Gold was issued a Certificate of Entry and Acceptance. Respondents contended that although Steamship Mutual is a P & I Club, it is not
Pioneer also issued receipts. When White Gold failed to fully pay its accounts, engaged in the insurance business in the Philippines. It is merely an association
Steamship Mutual refused to renew the coverage. of vessel owners who have come together to provide mutual protection against
liabilities incidental to shipowning.
Steamship Mutual thereafter filed a case against White Gold for collection of sum
of money to recover the unpaid balance. White Gold on the other hand, filed a Is Steamship Mutual engaged in the insurance business?
complaint before the Insurance Commission claiming that Steamship Mutual and A P & I Club is “a form of insurance against third party liability, where the third
Pioneer violated provisions of the Insurance Code. party is anyone other than the P & I Club and the members.” By definition then,
The Insurance Commission dismissed the complaint. It said that there was no Steamship Mutual as a P & I Club is a mutual insurance association engaged in
need for Steamship Mutual to secure a license because it was not engaged in the the marine insurance business.
insurance business and that it was a P & I club. Pioneer was not required to The records reveal Steamship Mutual is doing business in the country albeit
obtain another license as insurance agent because Steamship Mutual was not without the requisite certificate of authority mandated by Section 187 of the
engaged in the insurance business. Insurance Code. It maintains a resident agent in the Philippines to solicit
insurance and to collect payments in its behalf. Steamship Mutual even renewed

Page 21 of 47
its P & I Club cover until it was cancelled due to non-payment of the calls. Thus, Held: No. Petition granted.
to continue doing business here, Steamship Mutual or through its agent Pioneer,
Ratio:
must secure a license from the Insurance Commission.
According to the Insurance code, the Insurance Commissioner was authorized to
Since a contract of insurance involves public interest, regulation by the State is
suspend, directors, officers, and agents of insurance companies. In general, he
necessary. Thus, no insurer or insurance company is allowed to engage in the
was tasked to regulate the insurance business, which includes:
insurance business without a license or a certificate of authority from the
Insurance Commission. (2) The term "doing an insurance business" or "transacting an insurance
business," within the meaning of this Code, shall include
2. Pioneer is the resident agent of Steamship Mutual as evidenced by the
certificate of registration issued by the Insurance Commission. It has been (a) making or proposing to make, as insurer, any insurance contract;
licensed to do or transact insurance business by virtue of the certificate of
authority issued by the same agency. However, a Certification from the (b) making, or proposing to make, as surety, any contract of suretyship as a
Commission states that Pioneer does not have a separate license to be vocation and not as merely incidental to any other legitimate business or activity
an agent/broker of Steamship Mutual. of the surety; (c) doing any kind of business, including a reinsurance business,
specifically recognized as constituting the doing of an insurance business within
Although Pioneer is already licensed as an insurance company, it needs a the meaning of this Code; (d) doing or proposing to do any business in
separate license to act as insurance agent for Steamship Mutual. Section 299 of substance equivalent to any of the foregoing in a manner designed to evade the
the Insurance Code clearly states: provisions of this Code. (Insurance Code, Sec. 2[2])
SEC. 299 No person shall act as an insurance agent or as an insurance broker in The contract of agency between Philamlife and its agents wasn’t included with
the solicitation or procurement of applications for insurance, or receive for the Commissoner’s power to regulate the business. Hence, the
services in obtaining insurance, any commission or other compensation from any Insurance commissioner wasn’t vested with jurisidiction under the rule “expresio
insurance company doing business in the Philippines or any agent thereof, unius est exclusionalterius”.
without first procuring a license so to act from the Commissioner…
The respondent contended that the commissioner had the quasi-judicial power
Philam v Arnaldo G.R. No. 76452 July 26, 1994 to adjudicate under Section 416 of the Code. It stated:

The Commissioner shall have the power to adjudicate claims and complaints
Facts:
involving any loss, damage or liability for which an insurer may be answerable
One Ramon Paterno complained about the unfair practices committed by the under any kind of policy or contract of insurance, or for which such insurer may
company against its agents, employees and consumers. be liable under a contract of suretyship, or for which a reinsurer may be used
The Commissioner called for a hearing where Paterno was required to specify under any contract or reinsurance it may have entered into, or for which a mutual
which acts were illegal. Paterno then specified that the fees and charges stated benefit association may be held liable under the membership certificates it has
in the Contract of Agency between Philam and its agents be declared void. issued to its members, where the amount of any such loss, damage or liability,
Philam, on the other hand, averred that there Paterno must submit a verified excluding interest, costs and attorney's fees, being claimed or sued upon any
formal complaint and that his letter didn’t contain information Philam was kind of insurance, bond, reinsurance contract, or membership certificate does
seeking from him. Philam then questioned the Insurance Commission’s not exceed in any single claim one hundred thousand pesos.
jurisdiction over the matter and submitted a motion to quash.
This was, however, regarding complaints filed by the insured against the
The commissioner denied this. Hence this petition. Insurance company.
Issue: Whether or not the resolution of the legality of the Contract of Agency Also, the insurance code only discusses the licensing requirements for agents
falls within the jurisdiction of the Insurance Commissioner.
and brokers. The Insurance Code does not have provisions governing the
relations between insurance companies and their agents.
Page 22 of 47
Investment Planning Corporation of the Philippines v. Social Security Since World War I, the determination of enemy nationality of corporations has
Commission- “that an insurance company may have two classes of agents who been discussed in many countries, belligerent and neutral. A corporation was
sell its insurance policies: (1) salaried employees who keep definite hours and subject to enemy legislation when it was controlled by enemies, namely managed
work under the control and supervision of the company; and (2) registered under the influence of individuals or corporations themselves considered as
representatives, who work on commission basis.” enemies…
The Philippine Insurance Law (Act No 2427, as amended), in Section 8, provides
The agents under the 2nd sentence are governed by the Civil Code laws on that “anyone except a public enemy may be insured”. It stands to reason that
agency. This means that the regular courts have jurisdiction over this category. an insurance policy ceases to be allowable as soon as an insured becomes a
public enemy.
The respondent having an enemy corporation on December 10, 1941, the
insurance policy issued in its favor on October 1, 1941, by the petitioner had
FILIPINAS COMPANIA DE SEGUROS vs. CHRISTERN HUENEFELD and ceased to be valid and enforceable, and since the insured good were burned
CO., INC. 89 Phil 54 during the war, the respondent was not entitled to any indemnity under said
policy from the petitioner. However, elementary rule of justice (in the absence
FACTS: of specific provisions in the Insurance Law) require that the premium paid by the
respondent for the period covered by its policy from December 11, 1941, should
be returned by the petitioner.
On October 1, 1941, the respondent corporation, Christern Huenefeld and Co.,
Inc., after payment of corresponding premium, obtained from the petitioner,
Filipinas Cia de Seguros fire policy covering merchandise contained in a building
located at Binondo, Manila. On February 27, 1942 or during the Japanese military Sales De Gonzaga V. Crown Life Insurance Co.
occupation, the building and insured merchandise were burned. In due time, the
respondent submitted to the petitioner its claim under the policy. The petitioner FACTS:
refused to pay the claim on the ground that the policy in favor of the respondent
that ceased to be a force on the date the United States declared war against
Germany, the respondent corporation (through organized under and by virtue of  September 26, 1939: Crown Life Insurance Co. whose home office is based
the laws of Philippines) being controlled by German subjects and the petitioner in Toronto, Canada issued to Ramon Gonzaga through its branch office in
being a company under American jurisdiction when said policy was issued on Manila a 20-year endowment policy for P15,000 which had an annual
October 1, 1941. The theory of the petitioner is that the insured merchandise
was burned after the policy issued in 1941 had ceased to be effective because premium of P591.
the outbreak of the war between United States and Germany on December 10,  Payment was only until September 6, 1941 because of the outbreak of the
1941, and that the payment made by the petitioner to the respondent
war since Crown is an enemy corp. order to be closed during the Japanese
corporation during the Japanese military occupation was under pressure.
ISSUE: occupation. However, despite that it offered a privilege to accept premium
payments in the place of its employee in Ermita but of which Gonzaga did
Whether or not the respondent corporation is a corporation of public enemy. not avail.
RULING:  Through the automatic premium loan clause, it continued until June 12, 1943
 May 1, 1945: It reopened but still Gonzaga did not pay although there was a
Since the majority of stockholders of the respondent corporation were German reinstatement clause providing certain conditions within three years from the
subjects, the respondent became an enemy of the state upon the outbreak of
the war between US and Germany. The English and American cases relied upon date of lapse on application of the insured
by the Court of Appeals lost in force upon the latest decision of the Supreme  June 27, 1945: Gonzaga died from an accident
Court of US in which the control test has adopted.  Crown refused to pay because of the lapse of premium payment

Page 23 of 47
 RTC: against Gonzaga  RTC: it is a loan with equitable mortgage so the insurance proceeds should
ISSUE: W/N Gonzaga's widow can claim despite the absence of premium be credited to the loan and refund the overpayment.
payment during the outbreak of the war ISSUE: W/N Cosio as mortgagee is entitled to the insurance proceeds for her
own benefit
HELD: NO. Affirmed
HELD: YES. Modify. collection of insurance proceeds shall not be deemed to
 Non-payment at the day involves absolute forfeiture is such be the terms of have compensated the obligation of the Palileo to Cosio, but bars the Cosio from
the contract claiming its payment from the Palileo; and Cosio shall pay to Palileo P810
 failure to notify the postal address during the war is not an excuse representing the overpayment made by Palileo by way of interest on the loan.
 There is no duty when the law forbids and there is no obligation without  When the the mortgagee may insure his interest in the property
corresponding right enjoyed by another independently of the mortgagor , upon the destruction of the property the
 opening of an interim office partook of the nature of the privilege to the policy insurance money paid to the mortgagee will not inure to the benefit of the
holders to keep their policies operative rather than a duty to them under the mortgagor, and the amount due under the mortgage debt remains
contract unchanged. The mortgagee, however, is not allowed to retain his claim
against the mortgagor, but it passes by subrogation to the insurer, to the
WEEK 5 extent of the insurance money paid
 It is true that there are authorities which hold that "If a mortgagee procures
Palileo V. Cosio (1955) insurance on his separate interest at his own expense and for his own benefit,
without any agreement with the mortgagor with respect thereto, the
FACTS:
mortgagor has no interest in the policy, and is not entitled to have the
insurance proceeds applied in reduction of the mortgage debt" But these
 Cherie Palileo (debtor-mortgagor) filed a complaint against Beatriz Cosio
authorities merely represent the minority view
(creditor-mortgagee) praying that their transaction be one of a loan with an
equitable mortgage to secure the payment of the loan. The original counsel
San Miguel Brewery V. Law Union And Rock Insurance
of Cosio Atty. Guerrero being appointed Undersecretary of Foreign Affairs so
she forgot the date of the trial and she was substituted.
 it is a loan of P12,000 secured by a "Conditional Sale of Residential Building" Lessons Applicable:
with right to repurchase. After the execution of the contract, Cosio insured in  Mortgagor (Insurance)
her name the building with Associated Insurance & Surety Co. against fire.  Measure of Insurable Interest (Insurance)
 The building was partly destroyed by fire so she claimed an indemnity of  Effect of Change of Interest in Thing Insured (Insurance)
P13,107  Effect of transfer of thing insured (Insurance)
 Palileo demanded that the amount of insurance proceeds be credited to her Laws Applicable: sec. 16,sec. 19 (now sec. 20),sec. 50,sec.55 (now sec. 58) of
loan the Insurance Code (all old law)

Page 24 of 47
 the mere transfer of a thing insured does not transfer the policy, but
FACTS: suspends it until the same person becomes the owner of both the policy and
 In the contract of mortgage, the owner P.D. Dunn had agreed, at his own the thing insured
expense, to insure the mortgaged property for its full value and to indorse  Undoubtedly these policies of insurance might have been so framed as to
the policies in such manner as to authorize the Brewery Company to receive have been "payable to the San Miguel Brewery, mortgagee, as its interest
the proceeds in case of loss and to retain such part thereof as might be may appear, remainder to whomsoever, during the continuance of the risk,
necessary to satisfy the remainder then due upon the mortgage debt. may become the owner of the interest insured." (Sec 54, Act No. 2427.) Such
Instead, however, of effecting the insurance himself Dunn authorized and a clause would have proved an intention to insure the entire interest in the
requested the Brewery Company to procure insurance on the property in the property, not merely the insurable interest of the San Miguel Brewery, and
amount of P15,000 at Dunn's expense. would have shown exactly to whom the money, in case of loss, should be
 San Miguel insured the property only as mortgagee. paid. But the policies are not so written.
 Dunn sold the propert to Henry Harding. The insurance was not assigned by  The blame for the situation thus created rests, however, with the Brewery
Dunn to Harding. rather than with the insurance companies, and there is nothing in the record
 When it was destroyed by fire, the two companies settled with San Miguelto to indicate that the insurance companies were requested to write insurance
the extent of the mortgage credit. upon the insurable interest of the owner or intended to make themselves
 RTC: Absolved the 2 companies from the difference. Henry Harding is liable to that extent
not entitled to the difference between the mortgage credit and the face value  If by inadvertence, accident, or mistake the terms of the contract were not
of the policies. fully set forth in the policy, the parties are entitled to have it reformed. But
 Henry Harding appealed. to justify the reformation of a contract, the proof must be of the most
ISSUE: satisfactory character, and it must clearly appear that the contract failed to
1. W/N San Miguel has insurable interest as mortgagor only to the extent of the express the real agreement between the parties
mortgage credit - YES  In the case now before us the proof is entirely insufficient to authorize
2. W/N Harding has insurable interest as owner - NO reformation.

Gonzalez Lao v. Yek Tong Lin Fire & Marine Insurance - Insurance
HELD: affirmed
 section 19 of the Insurance Act: Premiums
 a change of interest in any part of a thing insured unaccompanied by a
Facts:
corresponding change of interest in the insurance, suspends the insurance to
an equivalent extent, until the interest in the thing and the interest in the
> Gonzales was issued 2 fire insurance policies by Yek for 100T covering his leaf
insurance are vested in the same person
tobacco prducts.
 section 55:

Page 25 of 47
the property or properties consisting of stocks in trade, goods in process and/or
> They were stored in Gonzales’ building on Soler St., which on Jan. 11, 1928,
inventories only hereby insured, xxx
burned down.
The petitioners’ stocks were destroyed by fire. He then filed a claim which was
> Art. 3 of the Insurance policies provided that: “Any insurance in force upon subsequently denied because the petitioner’s stocks were covered by two other
fire insurance policies issued by PFIC. The basis of the private respondent's
all or part of the things unsured must be declared in writing by the insured and
denial was the petitioner's alleged violation of Condition 3 of the policy. The
he (insured) should cause the company to insert or mention it in the policy. Insurance Commission found that the petitioner did not violate Condition 3 as he
had no knowledge of the existence of the two fire insurance policies obtained
Without such requisite, such policy will be regarded as null and void and the
from the PFIC; that it was Cebu Tesing Textiles which procured the PFIC policies
insured will be deprived of all rights of indemnity in case of loss.” w/o informing him or securing his consent; and that Cebu Tesing Textile, as his
creditor, had insurable interest on the stocks.
> Notwithstanding said provision, Gonzales entered into other insurance
Issue: Whether or not Geagonia is prohibited from recovering from the Country
contracts. When he sought to claim from Yek after the fire, the latter denied any
Bankers ?
liability on the ground of violation of Art. 3 of the said policies.
Held: A policy may declare that a violation of specified provisions thereof shall
> Gonzales however proved that the insurer knew of the other insurance policies avoid it,otherwise, the breach of an immaterial provision does not avoid the
policy. To constitute a violation of the “other insurance” clause, the other
obtained by him long efore the fire, and the insurer did NOT rescind the insurance
insurance must be upon the same subject matter, the same interest therein, and
polices in question but demanded and collected from the insured the premiums. the same risk

Issue:

Saura Import & Export Co., Inc. V. Philippine International Surety Co.,
Whether or not Yek is still entitled to annul the contract.
Inc. (1963)
Held:
Lessons Applicable: Mortgagor (Insurance)
NO. Laws Applicable:

The action by the insurance company of taking the premiums of the insured
FACTS:
notwithstanding knowledge of violations of the provisions of the policies
amounted to waiver of the right to annul the contract of insurance.  Saura Import & Export Co Inc., mortgaged to the Phil. National Bank, a
parcel of land.
 The mortgage was amended to guarantee an increased amount, bringing
Geagonia vs. Court of Appeals, 241 SCRA 152
the total mortgaged debt to P37,000
Facts: Geagonia, owner of a store, obtained from Country Bankers 1year fire  On the land mortgage is a building owned by Saura Import & Export Co Inc.
insurance covering the stock trading of dry goods. The policy noted the
which was insured with Philippine International Surety (Insurer) even
requirement that"
3. The insured shall give notice to the Company of any insurance or insurances before the mortgage contract so it was required to endorse to mortgagee
already effected, or which may subsequently be effected, covering any of PNB
Page 26 of 47
 October 15, 1954: Barely 13 days after the issuance of the fire insurance due date, the policy would be in default, and if the premium remained unpaid
until the end of the grace period, the policy would automatically lapse and
policy, the insurer cancelled it. Notice of the cancellation was given to PNB become void.
(mortgagee). But Saura (insured) was not informed.
Eulogio paid the premiums, however he failed to pay the premium due on
 April 6, 1955: The building and all its contents worth P40,685.69 were
January 24, 1998, even after the lapse of the grace period of 31 days. Therefore,
burned so Saura filed a claim with the Insurer and mortgagee Bank lapsed and become void. Eulogio submitted to the Cabanatuan District Office of
 RTC: dismissed Insular Life an application for reinstatement together with the payment of the
premium due on January 24. Insular Life notified Eulogio that his application for
ISSUE: W/N Philippine International Surety should be held liable for the claim
reinstatement could not be fully processed because of the unpaid interest
because notice to only the mortgagee is not substantial thereon. Eulogio was likewise advised by Malaluan (insurance agent) to pay the
premiums that subsequently became due April 1998 and July 1998, plus interest.
HELD:YES. Appealed from is hereby reversed. Philippine International Surety
September 17, 1998. Eulogio went to Malaluan's house and paid for the interest
Co., Inc., to pay Saura Import & Export Co., Inc., P29,000 which was received by Malaluan's husband. Later that day, Eulogio died. Without
 It was the primary duty of Philippine International Surety to notify the the knowledge of Eulogio's death, Malaluan forwarded to the Insular Life the
application for reinstatement and the payment made by Eulogio. However,
insured, but it did not Insular Life did not act upon such reinstatement for they knew already of
 If a mortgage or lien exists against the property insured, and the policy Eulogio's death.
contains a clause stating that loss, if any, shall be payable to such
September 28, 1998, Violeta filed for the insurance claim. Insular Life then
mortgagee or the holder of such lien as interest may appear, notice of informed Violeta in a letter that her claim could not be processed because the
cancellation to the mortgagee or lienholder alone is ineffective as a insurance policy had lapsed already and that Eulogio failed to reinstate the same
and the payment made done thru Malaluan's husband was, under the insurance
cancellation of the policy to the owner of the property.
policy, was considered a deposit only until approval of the said application.
 liability attached principally the insurance company, for its failure to give Enclosed to this letter was a check representing the full refund of the past
notice of the cancellation of the policy to Saura payments made by Eulogio, amounting to P25,417.
 it is unnecessary to discuss the errors assigned against appellee bank
Violeta requested for a reconsideration of her claim and returned the check to
Insular Life. Insular Life agreed to conduct a re-evaluation of Violeta's claim.
PNB V CA Without waiting for the result of the re-evaluation, Violeta filed with the RTC a
complaint for death claim benefit alleging the Insular Life was engaged in unfair
Violeta Lalican claim settlement practice and deliberately failed to act with reasonable
vs The Insular Life Insurance Company promptness on her insurance claim. Violeta claims for the P1.5M insurance, plus
Ponente: Chico-Nazario interest, attorney's fees and cost of suit.

Facts: Insular Life filed with the RTC an answer with counterclaim saying that the
Violeta is the widow of the Eulogio Lalican. During his lifetime, Eulogio applied insurance claim was rendered void due to non-payment of the premium and
for an insurance policy with Insular Life on April 24, 1997 which contained a 20- countered that Violeta should be ordered to pay attorney's fees and expenses of
year endowment variable income package flexi plan worth P500k with two riders litigation incurred by Insular Life.
worth P500k each. Violeta was named the primary beneficiary.
RTC declared that Violeta failed to establish by preponderance of evidence her
Under the terms, Eulogio was to pay premiums on a quarterly basin in the cause of action against the defendant. Violeta failed to establish that the receipt
amount of P8,062 with a grace period of 31 days for the payment of each of payment by Malaluan amounted to the reinstatement of the insurance policy.
premium subsequent to the first. If any premium was not paid on or before the
Page 27 of 47
Violeta filed for motion for reconsideration but was denied as well; hence she Posadas (Collector of Internal Revenue) upon showing by ElOriente that
elevated her case for review on Certiorari. such premiums were legitimate expenses of the business.U p o n t h e d e a t h o f
the manager, El Oriente received all the proceeds of the
Issues: (a) Whether the decision of the court can still be reviewed despite having l i f e insurance policy together with the interest and the dividends accruing
allegedly attained finality and despite the mode of appeal of Violeta erroneous. thereon, aggregatingP104,957.88. Posadas assessed and levied the sum
(b) Whether the RTC has decided the case on a question of law not in accord of P3,148.74 as income tax on theproceeds of the insurance policy,
with law and applicable decisions of the Supreme Court. which was paid by El Oriente under protest. El Orienteclaiming exemption
under Section 4 of the Income Tax Law.
Ruling:
Petition lacks merit.

RTC's decision has long acquired finality for Violeta failed to file a notice of appeal Issue:
more than five months after the decision was rendered. Whether or not the proceeds of insurance taken by a corporation on
the life of animportant official to indemnify it against loss in case of
As to the substantial claim of whether there is insurable interest, the Court says his death, are taxable as incomeunder the Philippine Income Tax Law?
that the matter of insurable interest is entirely irrelevant and the real point of Ruling:
contention herein is whether Eulogio was able to reinstate the lapsed insurance The Income Tax Law for the Philippines is Act No. 2833, as amended. In chapter
policy on his life before his death. IOn Individuals, is to be found section 4
which provides that, "The following incomes shallbe exempt from the provisions
of this law:
The Court rules in the negative, for the insurance policy is clear on the procedure (a) The proceeds of life insurance policies paid tobeneficiaries upon the death
of the reinstatement of the insurance contract, of which Eulogio has failed to of the insured ... ." The Chapter on Corporations does notprovide as above. It is
accomplish before his death. As provided by the policy, insurance shall be certain that the proceeds of life insurance policies are exempt. It isnot so
deemed reinstated upon the approval of the insurance policy of the application certain that the proceeds of life insurance policies paid to corporate
for reinstatement. The approval should be made during the lifetime of the beneficiariesupon the death of the insured are likewise exempt.T h e
insured, in the case at bar, it wasn’t. situation will be better elucidated by a brief reference to laws
on the samesubject in the United States. The Income Tax Law
El Oriente Fabrica de Tabacos, Inc. vs. Juan Posadas, Collector of Internal of 1916 extended to the PhilippineL e g i s l a t u r e , w h e n i t c
Revenue,G.R. No. 34774, September 21, 1931 ame to enact Act No. 2833, to copy the American
Topic: s t a t u t e . Subsequently, the Congress of the United States enacted
Insurable Interest in life and health (Section 10) its Income Tax Law of 1919, in which certain doubtful subjects were clarified.
Facts: Thus, as to the point before us, it was madeclear, when not only in the part of
the law concerning individuals were exemptions providedfor beneficiaries, but
Insurer: Manufacturers Life Insurance Co., of Toronto, Canada, also in the part concerning corporations, specific reference was
thru its local agent E.E.ElserInsured: A. Velhagen (manager of El madet o t h e e x e m p t i o n s i n f a v o r o f i n d i v i d u a l s , t h e r e b y
Oriente)Beneficiary: El Oriente Fabrica de Tabacos, Inc.El Oriente, in order to m a k i n g t h e s a m e a p p l i c a b l e t o corporations.
protect itself against the loss that it might suffer by reason of the death of its This was authoritatively pointed out and decided by the Unite
manager, whose death would be a serious loss to El Oriente procured fromthe d S t a t e s Supreme Court in the case of United States
Insurer an insurance policy on the life of the said manager for the sum of 50,000 vs
USD withElOriente as . Supplee-Biddle Hardware Co. ( [1924], 265U.S., 189), which involved facts
the designated sole beneficiary. The insured has no int quite similar to those before us.To quote the exact words in the cited case of
e r e s t o r participation in the proceeds of said life insurance policy.El Oriente Chief Justice Taft delivering the opinionof the court:It is earnestly pressed
charged as expenses of its business all the said premiums and deducted upon us that proceeds of life insurance paid on the death of the insured
the same from its gross incomes as reported in its are in fact capital, and cannot be taxed as income … that proceeds of al i f e
a n n u a l i n c o m e t a x r e t u r n s , w h i c h deductions were allowed by insurance policy paid on the death of the insured are not
Page 28 of 47
usually proceeds of life insurance policies paid to beneficiaries upon the death of the
c l a s s e d a s income.C o n s i d e r i n g , t h e r e f o r e , t h e p u r p o r t o f t h insured . . ." Section 10, as amended, in Chapter II On Corporations, provides
e s t i p u l a t e d f a c t s , c o n s i d e r i n g t h e uncertainty of Philippine law, that, "There shall be levied, assessed, collected, and paid annually upon the total
and considering the lack of express legislative intention to taxthe proceeds of life net income received in the preceding calendar year from all sources by every
insurance policies paid to corporate beneficiaries, particularly when inthe corporation . . .a tax of three per centum upon such income . . ." Section 11 in
exemption in favor of individual beneficiaries in the chapter on this subject, the same chapter, provides the exemptions under the law, but neither here nor
the clause isi n s e r t e d " e x e m p t f r o m t h e in any other section is reference made to the provisions of section 4 in Chapter
provisions of this law," we deem it reasonable to hold thepro I.
ceeds of the life insurance policy in question as representing
a n i n d e m n i t y a n d n o t taxable Under the view we take of the case, it is sufficient for our purposes to direct
income.T h e f o r e g o i n g p r o n o u n c e m e n t w i l l r e s u l t i n t h e j u d g m e attention to the anomalous and vague condition of the law. It is certain that the
n t b e i n g r e v e r s e d a n d i n another judgment being rendered in favor of proceeds of life insurance policies paid to individual beneficiaries upon the death
El Oriente. of the insured are exempt. It is not so certain that the proceeds of life insurance
policies paid to corporate beneficiaries upon the death of the insured are likewise
exempt. But at least, it may be said that the law is indefinite in phraseology and
El Oriente v. Posadas - Taxability of Insurance Proceeds does not permit us unequivocally to hold that the proceeds of life insurance
56 PHIL 147 (1931) policies received by corporations constitute income which is taxable

Facts: It will be recalled that El Oriente, took out the insurance on the life of its
> El Oriente in order to protect itself against the loss that it might suffer by manager, who had had more than thirty-five years' experience in the
reason of the death of its manager, A. Velhagen, who had had more than thirty- manufacture of cigars in the Philippines, to protect itself against the loss it might
five (35) years of experience in the manufacture of cigars in the Philippines, suffer by reason of the death of its manager. We do not believe that this fact
procured from the Manufacturers Life Insurance Co., of Toronto, Canada, thru signifies that when the plaintiff received P104,957.88 from the insurance on the
its local agent E. E. Elser, an insurance policy on the life of the said A. Velhagen life of its manager, it thereby realized a net profit in this amount. It is true that
for the sum of $50,000, United States currency designating itself as the the Income Tax Law, in exempting individual beneficiaries, speaks of the
beneficiary. proceeds of life insurance policies as income, but this is a very slight indication
> El Oriente paid for the premiums due thereon and charged as expenses of its of legislative intention. In reality, what the plaintiff received was in the nature of
business all the said premiums and deducted the same from its gross incomes an indemnity for the loss which it actually suffered because of the death of its
as reported in its annual income tax returns, which deductions were allowed upon manager.
a showing that such premiums were legitimate expenses of its business.
> Upon the death of A. Velhagen in 1929, the El Oriente received all the
proceeds of the said life insurance policy, together with the interests and the
dividends accruing thereon, aggregating P104,957.88
> CIR assessed El Oriente for deficiency taxes because El Oriente did not include
as income the proceeds received from the insurance.
Insular Life vs. Ebrado
Issue: 80 SCRA 181
Whether or not the proceeds of insurance taken by a corporation on the life of Facts:
an important official to indemnify it against loss in case of his death, are taxable > Buenaventura Ebrado was issued al life plan by Insular Company. He
as income under the Philippine Income Tax Law designated Capriona as his beneficiary, referring to her as his wife.
> The insured then died and Carponia tried to claim the proceeds of the said
Held: plan.
NOT TAXABLE. > She admitted to being only the common law wife of the insured.
In Chapter I of the Tax Code, is to be found section 4 which provides that, "The > Pascuala, the legal wife, also filed a claim asserting her right as the legal
following incomes shall be exempt from the provisions of this law: (a) The wife. The company then filed an action for interpleader.
Page 29 of 47
special laws.Matters not expressly provided for in such special laws shall
Issue: be regulated by this Code.”Article 2012 of the same Code states that, “any
person who is forbidden from receiving any donation under Article 739 cannot
Whether or not the common law wife named as beneficiary can collect the be named beneficiary of a life insurance policy by the person who cannot make
proceeds. a donation to him.” Therefore, common-law spouses are barred from
receivingdonations from each other.Article 739 provides:“The following
Held: donations shall be void:Those made between persons who were guilty of adultery
NO. or concubinage at the time of thedonation;Those made between persons found
The civil code prohibitions on donations made between persons guilty of guilty of the same criminal offense, in consideration thereof;Those made to a
adulterous concubinage applies to insurance contracts. On matters not public officer or his wife, descendants or ascendants by reason of his office.
specifically provided for by the Insurance Law, the general rules on Civil law shall
apply. A life insurance policy is no different from a civil donation as far as the In the case referred to in No. 1, the action for declaration of nullity may be
beneficiary is concerned, since both are founded on liberality. brought by thespouse of the donor or donee; and the guilt of the donee may be
proved by preponderance ofevidence in the same action.”In essence, a life
DOCTRINE:General rules of civil law should be applied to resolve any void in the insurance policy is no different from a civil donation insofar as the beneficiary
Insurance Law, aspronounced in Article 2011 of the NCC.A contract of insurance isconcerned. Both are founded upon the same consideration: liberality. As a
is personal in character.FACTS:Buenaventura Cristor Ebrado was married to consequence, theproscription in Art. 739 of the New Civil Code should equally
Pascuala Ebrado. During his lifetime, he was livingwith his common-law wife, operate in life insurance contracts.In the case at bar, the requisite proof of
Carponia Ebrado, although he was not legally separated from his legalwife. common-law relationship between the insured and thebeneficiary has
Buenaventura was issued by The Insular Life Assurance Co., Ltd., Policy No. been supplied by the stipulations between the parties in the pre-trial conference.
009929 on a whole-life plan for PhP 5,8882.00 with a rider for Accidental Death Itwas agreed an stipulated that the deceased insured Buenaventura Ebrado was
Benefits for the same amount.Buenaventura designated Carponia Ebrado as the married toPascuala Ebrado and that, during the lifetime of the deceased insured,
revocable beneficiary in his policy. Buenaventura died as a result of an accident he was living with hiscommon-law wife, Carponia Ebrado. Based on the
when he was hit by a falling branch of a tree. As the insurance policy was still in foregoing, Carponia Ebrado is hereby declareddisqualified to be the beneficiary
force, The Insular Life Assurance Co., Ltd stands liable to pay the coverage. of the late Buenaventura Ebrado in his life insurance policy. Theproceeds of the
Carponia Ebrado, his common-law wife, filed with the insurer a claim for the policy are hereby held payable to the estate of the deceased insured.
proceeds of the policy as the designated beneficiary therein. Pascuala Vda. de
Ebrado also filed her claim as the widowof the deceased insured. She asserts
that she is the one entitled to the insurance proceeds, not the common-law wife. Vda. de Consuegra vs. GSIS, DPWH, Rosario Diaz 37 SCRA 315 [1971]MAIN
In doubt as to whom the insurance proceeds shall be paid, the insurer, The POINT:
Insular Life AssuranceCo., Ltd, commenced an action for Interpleader before the FACTS:
CFI of Rizal. The late Jose Consuegra, at the time of his death, was employed as a shop foreman of DPWH
in the province ofSurigao del Norte. In his lifetime, he contracted 2 marriages, the first with
ISSUE: Can a common-law wife of a man who was not legally separated from respondent Rosario Diaz, solemnized in the parish church of San Nicolas de Tolentino, Surigao,
his legal wife be a beneficiary of his life insurance plan? on July 15, 1937 bearing 2 children, Jose Consuegra, Jr. and Pedro Consuegra, but both
predeceased their father; and the second with herein petitioner Basilia Berdin, which
HELD/RULING was contracted in good faith while the first marriage was subsisting, on May 1, 1957 in the
No.The Insurance Act (RA 2327, as amended) or even the new Insurance Code same parish and municipality, out of which marriage
(PD No. 612, asamended) does not contain any specific provision grossly were born seven children, namely, Juliana, Pacita, Maria Lourdes, Jose, Rodrigo,
resolutory to the question at hand.Section 50 of the Insurance Act, which Lenida and Luz *(Luis) all surnamedConsuegra.When Consuegra died the proceeds of
provides that “(t)he insurance shall be applied exclusively tothe proper interest his life insurance were paid by the GSIS to petitioner Basilia Berdin and her children who were
of the person in whose name it is made,” cannot be interpreted that it includesthe the beneficiaries named in the policy. Having been in the service of the government for
beneficiary because a contract of insurance is personal in character. The general 22.5028 years, Consuegra was entitled to retirement insurance benefits in the sum of P6,
rules of civillaw should be applied to resolve this void in the Insurance Law.Article 304.47, however, he did not designate any beneficiary who would receive the retirement
2011 of the New Civil Code states: “The contract of insurance is governed by insurance benefits due to him. Respondent Diaz, the widow (1st), filed a claim with the GSIS
Page 30 of 47
asking that the retirement insurance benefits be paid to her as the only legal heir. Petitioner application for retirement insurance. The beneficiary of the retirement insurance can only claim
Berdin and her children, likewise, filed a similar claim with the GSIS, asserting that being the the proceeds of the retirement insurance if the employee dies before retirement.
beneficiaries named in the life insurance policy,they are the only ones entitled If the employee failed or overlooked to state the beneficiary of his retirement
to receive the retirement insurance benefits due the deceased. GSIS: Diaz is entitled to insurance, the retirement benefits will accrue to his estate and will be given to
one-half, or 8/16, of the retirement insurance benefits while Berdin and their 7 children are his legal heirs in accordance with law, as in the case of a life insurance if no
entitled to the remaining one-half, or 8/16, each of them to receive an equal share of beneficiary is named in the insurance policy.
1/16.Dissatisfied, Berdin and her children filed a petition for mandamus with preliminary
injunction in the CFI of Surigao, against the respondents praying that they be declared the B.BOTH.
legal heirs and exclusive beneficiaries of the retirement insurance. CFI confirmed the action of The proceeds of the retirement insurance of the late Jose Consuegra should be divided equally
GSIS. Hence the present appeal by herein petitioners-appellants, Basilia Berdin and her between Rosario Diaz and his second wife Basilia Berdin and his children by her. The fact that
children contending that because the deceased failed to designate the beneficiaries in his the second marriage was contracted in good faith, the only just and equitable solution
retirement insurance, they being beneficiaries named in the life insurance should automatically in this case would be to recognize the right of the second wife to her share of
be considered the beneficiaries to receive the retirement insurance benefits, to the exclusion one-half in the property acquired by her and her husband, and consider the other
of Diaz. half as pertaining to the conjugal partnership of the first marriage.
ISSUES: The decision appealed from is affirmed
(a) Is the contention tenable?(b) To whom should this retirement insurance benefits be paid?
RULING:
A.NO. The contention of appellants is untenable.When Consuegra designated his beneficiaries Southert Luzon Employee’s Association v. Golpeo - Insurance
in his life insurance he could not have intended those beneficiaries of his lifeinsurance as also Beneficiaries
the beneficiaries of his retirement insurance because the provisions on retirement insurance
under theGSIS came about only when Com. Act 186 was amended by Rep. Act 660 on June 96 PHIL 83
16, 1951. Hence, it cannot be said that Facts:
because appellants were designated beneficiaries in Consuegra’s > SLEA is composed of laborers and employees of the LTBC and BTC (now BLTB
life insurance they automatically became the beneficiaries Co.), and one of its purposes is mutual aid of its members and their dependents
also of his retirement insurance. in case of death.
The provisions clearly indicate that there is need for the employee to file an > Roman Concepcion was a member until his death in 1950.
applicationfor retirement insurance benefits when he becomes a member of > In 1949, SLEA adopted a resolution providing that: A member may, if he
the GSIS, and he should state in his application thebeneficiary of his retirement chooses, put down his common law wife and/or children he had with her as his
insurance. Hence, the beneficiary named in the life insurance does not beneficiaries; and such person so named by the member will be the sole persons
automaticallybecome the beneficiary in the retirement insurance unless the same to be recognized by SLEA regarding claims for condolence contributions.
beneficiary in the life insurance is so designatedin the application for retirement > Roman listed as his beneficiaries Aquilina Maloles and their 4 children. After
insurance. his death, SLEA was able to collect voluntary contribution from its members
The GSIS offers two separate and distinct systems of benefits to its members amounting to P2,205.
paid out from two distinct and separate funds: > Three sets of claimants to the amount presented themselves to the association
(1) Life insurance- proceeds are paid to whoever is named the beneficiary in the namely:
life insurance policy as provided forin the Insurance Act (Act 2427, as amended), the o Juanita Golpeo, legal wife, and her children
beneficiary in a life insurance under the GSIS may not necessarily be an heir of the o Aquilina Maloles, the common law wife, and her children
insured. The insured in a life insurance may designate any person as beneficiary o Elsie Hicban, another common law wife of Roman, and her child.
unless disqualified to be so under the provisions of the Civil Code. And in the absence of > SLEA then filed an action for interpleader against the 3 conflicting claimants.
any beneficiary named in the life insurance policy, the proceeds of the insurance will > Trial court rendered a decision declaring Maloles and her children the sole
go to the estate of the insured. beneficiaries of the amount citing Del Val v. Del Val.
(2) Retirement insurance- primarily intended for the benefit of the employee to > Only Golpeo appealed. She argues that:
provide for his old age, or incapacity, after rendering service in the government for a > The insurance code does not apply since the association is not an insurance
required number of years. If the employee reaches the age of retirement, he gets the company but a mutual benefit association.
retirement benefits even to the exclusion of the beneficiary or beneficiaries named in his
Page 31 of 47
> The stipulation between SLEA and Roman was void for being contrary to law, was of age, but withheld the release of the shares of minors Karl Brian and Trisha
public morals and public policy, pursuant to Art. 739 of the CC ( donations Angelie pending submission of letters of guardianship. Insular alleged that the
between persons guilty of concubinage at the time of donation are void) complaint or petition failed to state a cause of action insofar as it sought to
declare as void the designation of Eva as beneficiary, because Loreto revoked
Issue: her designation as such in Policy No. A001544070 and it disqualified her in Policy
No. A001693029; and insofar as it sought to declare as inofficious the shares
Whether or not Golpeo, the legal wife is entitled to the amount. of Odessa, Karl Brian, and Trisha Angelie, considering that no settlement of
Loreto’s estate had been filed nor had the respective shares of the heirs been
Held: determined. Insular further claimed that it was bound to honor the insurance
NO. policies designating the children of Loreto with Eva as beneficiaries pursuant to
First of all, the lower court did not consider the association as a regular insurance Section 53 of the Insurance Code. Grepalife alleged that Eva was not designated
company, but merely ruled that the death benefit in question is analogous to as an insurance policy beneficiary; that the claims filed by Odessa, Karl Brian,
insurance. Besides, even the Administrative Code describes a mutual benefit and Trisha Angelie were denied because Loreto was ineligible for insurance due
company as one which provides any method of life insurance among its members to a misrepresentation in his application form that he was born on December 10,
out of dues or assessments collected from its membership. 1936 and, thus, not more than 65 years old when he signed it in September
2001; that the case was premature, there being no claim filed by the legitimate
Secondly, without considering the intimation in the brief for Maloles that Golpeo, family of Loreto; and that the law on succession does not apply where
by her silence and actions had acquiesced in the illicit relations between her the designation of insurance beneficiaries is clear.
husband and Maloles, Golpeo’s argument would certainly NOT apply to the ISSUE:
children of Maloles likewise named beneficiaries by the deceased. As a matter Whether or not illegitimate children can be beneficiaries in an insurance contract.
of fact, the NCC recognizes certain successional rights of illegitimate children. RULING:
Yes. Section 53 of the Insurance Code states that the insurance proceeds shall be
applied exclusively to the proper interest of the person in whose name or for
Heirs of Maramag v. Maramag whose benefit it is made unless otherwise specified in the policy. Pursuant
G.R. No. 181132 , June 5, 2009 thereto, it is obvious that the only persons entitled to claim the insurance
FACTS: proceeds are either the insured, if still alive; or the beneficiary, if the insured is
The case stems from a petition filed against respondents with the RTC already deceased, upon the maturation of the policy.The exception to this rule is
for revocation and/or reduction of insurance proceeds for being void and/or a situation where the insurance contract was intended to benefit third persons
inofficious. The petition alleged that: (1) petitioners were the legitimate wife and who are not parties to the same in the form of favorable stipulations or
children of Loreto Maramag (Loreto), while respondents were Loreto’s indemnity. In such a case, third parties may directly sue and claim from the
illegitimate family; (2) Eva de Guzman Maramag (Eva) was a concubine of Loreto insurer.
and a suspect in the killing of the latter, thus, she is disqualified to receive any Petitioners are third parties to the insurance contracts with Insular and
proceeds from his insurance policies from Insular Life Assurance Company, Ltd. Grepalife and, thus, are not entitled to the proceeds thereof. Accordingly,
(Insular) and Great Pacific Life Assurance Corporation (Grepalife) (3) the respondents Insular and Grepalife have no legal obligation to turn over the
illegitimate children of Loreto—Odessa, Karl Brian, and Trisha Angelie—were insurance proceeds to petitioners. The revocation of Eva as a beneficiary in one
entitled only to one-half of the legitime of the legitimate children, thus, the policy and her disqualification as such in another are of no moment considering
proceeds released to Odessa and those to be released to Karl Brian and Trisha that the designation of the illegitimate children as beneficiaries in Loreto’s
Angelie were inofficious and should be reduced; and (4) petitioners could not insurance policies remains valid. Because no legal proscription exists in naming
be deprived of their legitimes, which should be satisfied first. Insular admitted as beneficiaries the children of illicit relationships by the insured, the shares of
that Loreto misrepresented Eva as his legitimate wife and Odessa, Karl Brian, Eva in the insurance proceeds, whether forfeited by the court in view of
and Trisha Angelie as his legitimate children, and that they filed their claims for the prohibition on donations under Article 739 of the Civil Code or by the
the insurance proceeds of the insurance policies; that when it ascertained that insurers themselves for reasons based on the insurance contracts, must be
Eva was not the legal wife of Loreto, it disqualified her as a beneficiary and awarded to the said illegitimate children, the designated beneficiaries, to
divided the proceeds among Odessa, Karl Brian, and Trisha Angelie, as the the exclusion of petitioners. It is only in cases where the insured has not
remaining designated beneficiaries; and that it released Odessa’s share as she designated any beneficiary, or when the designated beneficiary is disqualified by
Page 32 of 47
law to receive the proceeds, that the insurance policy proceeds shall redound to
the benefit of the estate of the insured. Issues and Resolutions:
Philamcare brought the instant petition for review, raising the primary argument
that a health care agreement is not an insurance contract; hence the
Philamcare v. CA- Health Care Agreement "incontestability clause" under the Insurance Code Title 6, Sec. 48 does not
apply.
379 SCRA 356 (2002)
Facts: SC held that in the case at bar, the insurable interest of respondent's husband
> Ernani Trinos, applied for a health care coverage with Philamcare. In the in obtaining the health care agreement was his own health. The health care
standard application form, he answered NO to the following question: “Have you agreement was in the nature of non-life insurance, which is primarily a contract
or any of your family members ever consulted or been treated for high blood of indemnity. Once the member incurs hospital, medical or any other expense
pressure, heart trouble, diabetes, cancer, liver disease, asthma or peptic ulcer? arising from sickness, injury or other stipulated contingent, the health care
(If Yes, give details)” provider must pay for the same to the extent agreed upon under the contract.
> The application was approved for a period of one year from March 1, 1988 to
March 1, 1989. He was a issued Health Care Agreement, and under such, he Under the title Claim procedures of expenses, Philamcare. had 12 mos from the
was entitled to avail of hospitalization benefits, whether ordinary or emergency, date of issuance of the Agreement within which to contest the membership of
listed therein. He was also entitled to avail of "out-patient benefits" such as the patient if he had previous ailment of asthma, and six months from the
annual physical examinations, preventive health care and other out-patient issuance of the agreement if the patient was sick of diabetes or hypertension.
services. The periods having expired, the defense of concealment or misrepresentation no
> Upon the termination of the agreement, the same was extended for another longer lie.
year from March 1, 1989 to March 1, 1990, then from March 1, 1990 to June 1, Petitioner argues that respondent's husband concealed a material fact in his
1990. The amount of coverage was increased to a maximum sum of P75,000.00 application. It appears that in the application for health coverage, petitioners
per disability. required respondent's husband to sign an express authorization for any person,
> During the period of his coverage, Ernani suffered a heart attack and was organization or entity that has any record or knowledge of his health to furnish
confined at the Manila Medical Center (MMC) for one month beginning March 9, any and all information relative to any hospitalization, consultation, treatment or
1990. any other medical advice or examination.
> While her husband was in the hospital, Julita tried to claim the benefits under
the health care agreement. However, Philamcare denied her claim saying that Philamcare cannot rely on the stipulation regarding "Invalidation of agreement"
the Health Care Agreement was void. which reads:
> According to Philamcare, there was concealment regarding Ernani's medical Failure to disclose or misrepresentation of any material information by the
history. member in the application or medical examination, whether intentional or
Doctors at the MMC allegedly discovered at the time of Ernani's confinement that unintentional, shall automatically invalidate the Agreement from the very
he was hypertensive, diabetic and asthmatic, contrary to his answer in the beginning and liability of Philamcare shall be limited to return of all Membership
application form. Fees paid. An undisclosed or misrepresented information is deemed material if
> Julita had no choice but to pay the hospitalization expenses herself, its revelation would have resulted in the declination of the applicant by
amounting to about P76,000.00 Philamcare or the assessment of a higher Membership Fee for the benefit or
> After her husband was discharged from the MMC, he was attended by a benefits applied for.
physical therapist at home. Later, he was admitted at the Chinese General
Hospital (CGH). Due to financial difficulties, Julita brought her husband home The answer assailed by petitioner was in response to the question relating to the
again. In the morning of April 13, 1990, Ernani had fever and was feeling very medical history of the applicant. This largely depends on opinion rather than fact,
weak. Julita was constrained to bring him back to the CGH where he died on the especially coming from respondent's husband who was not a medical doctor.
same day. Where matters of opinion or judgment are called for, answers made in good faith
> Julita instituted, an action for damages against Philamcare. She asked for and without intent to deceive will not avoid a policy even though they are untrue.
reimbursement of her expenses plus moral damages and attorney's fees. RTC Thus,
decided in favor of Julita. CA affirmed.
Page 33 of 47
(A)lthough false, a representation of the expectation, intention, belief, opinion, bar, must be liberally construed in favor of the subscriber, and if doubtful or
or judgment of the insured will not avoid the policy if there is no actual fraud in reasonably susceptible of two interpretations the construction conferring
inducing the acceptance of the risk, or its acceptance at a lower rate of premium, coverage is to be adopted, and exclusionary clauses of doubtful import should
and this is likewise the rule although the statement is material to the risk, if the be strictly construed against the provider.
statement is obviously of the foregoing character, since in such case the insurer
is not justified in relying upon such statement, but is obligated to make further Nario V. Philippine American Life Insurance Co. Of Canada (1967)
inquiry. There is a clear distinction between such a case and one in which the G.R. No. L-22796 June 26, 1967
insured is fraudulently and intentionally states to be true, as a matter of Lessons Applicable: Irrevocable Designation (Insurance)
expectation or belief, that which he then knows, to be actually untrue, or the
impossibility of which is shown by the facts within his knowledge, since in such FACTS:
case the intent to deceive the insurer is obvious and amounts to actual fraud. June 12, 1959: Philippine American Life Insurance Co. issued a life insurance
The fraudulent intent on the part of the insured must be established to warrant to Mrs. Alejandra Santos-Mario a life insurance policy under a 20-year
rescission of the insurance contract. Concealment as a defense for the health endowment plan, with a face value of P5,000 designating her husband Delfin
care provider or insurer to avoid liability is an affirmative defense and the duty Nario and their unemancipation son Ernesto Nario, as her irrevocable
to establish such defense by satisfactory and convincing evidence rests upon the beneficiaries
provider or insurer. In any case, with or without the authority to investigate, June, 1963: She submitted her loan application to the life insurance co. with
petitioner is liable for claims made under the contract. Having assumed a signature of her husband in two capacities:
responsibility under the agreement, petitioner is bound to answer the same to irrevocable beneficiaries
the extent agreed upon. In the end, the liability of the health care provider father-guardian of minor irrevocable beneficiary Ernesto
attaches once the member is hospitalized for the disease or injury covered by Insurance Co. denied asking that the legal guardian must be authorized by the
the agreement or whenever he avails of the covered benefits which he has court in a competent guardianship proceeding
prepaid. Upon denial, she opted to surrender her insurance policy in exchange of its cash
Under Section 27 of the Insurance Code, "a concealment entitles the injured surrender value of P520 but it was also denied on the same ground
party to rescind a contract of insurance." The right to rescind should be exercised September 10, 1963: Mrs. Alejandra Santos-Nario and her husband, Delfin
previous to the commencement of an action on the contract. In this case, no Nario, brought suit against the Philippine American Life Insurance Co
rescission was made. Besides, the cancellation of health care agreements as in RTC: favored the insurance company
insurance policies require the concurrence of the following conditions: CA: vested interest or right of the beneficiaries in the policy should be measured
Prior notice of cancellation to insured; on its full face value and not on its cash surrender value, for in case of death of
Notice must be based on the occurrence after effective date of the policy of one the insured, said beneficiaries are paid on the basis of its face value and in case
or more of the grounds mentioned; the insured should discontinue paying premiums, the beneficiaries may continue
Must be in writing, mailed or delivered to the insured at the address shown in paying it and are entitled to automatic extended term or paid-up insurance
the policy; options, etc. and that said vested right under the policy cannot be divisible at
Must state the grounds relied upon provided in Section 64 of the Insurance Code any given time. policy loan and surrender of policy constitute acts of disposition
and upon request of insured, to furnish facts on which cancellation is based. or alienation of property rights and not merely of management or administration
because they involve the incurring or termination of contractual obligations
None of the above pre-conditions was fulfilled in this case. When the terms of ISSUE: W/N parents as guardians can enter into transactions for the benefit
insurance contract contain limitations on liability, courts should construe them in of minor irrevocable beneficiaries.
such a way as to preclude the insurer from non-compliance with his obligation.
Being a contract of adhesion, the terms of an insurance contract are to be
construed strictly against the party which prepared the contract — the insurer. HELD: NO. Affirmed.
By reason of the exclusive control of the insurance company over the terms and SEC. 7. Parents as guardians. — When the property of the child under parental
phraseology of the insurance contract, ambiguity must be strictly interpreted authority is worth two thousand pesos or less, the father or the mother, without
against the insurer and liberally in favor of the insured, especially to avoid the necessity of court appointment, shall be his legal guardian. When the
forfeiture. This is equally applicable to Health Care Agreements. The property of the child is worth more than two thousand pesos, the father or the
phraseology used in medical or hospital service contracts, such as the one at mother shall be considered guardian of the child's property, with the duties and
Page 34 of 47
obligations of guardians under these rules, and shall file the petition required by > West Coast Life Insurance Company issued two policies of insurance on the
Section 2 hereof. For good reasons the court may, however, appoint another life of Esperanza Villanueva, one for 2T, maturing April 1, 1943; and other for 3T
suitable person. maturing Mar. 31, 1943.
even if worth less than P2,000 parent's authority over the estate of the ward as > In both policies, West agreed to pay 2T either to Esperanza if still living on
a legal-guardian would not extend to acts of encumbrance or disposition, as Apr 1, 1943; or to beneficiary Bartolome Villanueva, or the father of the insured
distinguished from acts of management or administration. immediately upon receipt of the proof of death of Esperanza.
> The policy also gave her the right to change the beneficiary.
Villanueva V. Oro (1948) > In 1940, Bartolome died, and he was substituted as beneficiary under the
G.R. No. L-2227 August 31, 1948 policies by Mariano, Esparanza’s brother.
Lessons Applicable: Insured Outlives Policy (Insurance) > Esperanza died in 1944 without having collected the insurance
Laws Applicable: proceeds. Adverse claims for the proceeds were presented by the estate of
Esperanza on one hand and by Mariano on the other.
> CFI held that the estate of Esperanza was entitled to the proceeds to the
FACTS: exclusion of the beneficiary.
West Coast Life Insurance Company issued 2 policies of insurance on the life of
Esperanza J. Villanueva: Issue:
2,000 php - maturing on April 1, 1943 Whether or not the beneficiary is entitled to the proceeds.
if living, on the 1st day of April 1943 - to insured
upon death during the continuance of this policy - to the beneficiary Bartolome Held:
Villanueva, father of the insured, with right on the part of the insured to change NO.
the beneficiary Under the policies, the insurer obligated itself to pay the insurance proceeds to:
1940: Bartolome Villanueva died, Mariano J. Villanueva duly substituted as (1) the insured if the latter lived on the dates of maturity; or (2) the beneficiary
beneficiary, a brother of the insured if the insured died during the continuance of the policies. The first contingency
3,000 php - maturing on March 31, 1943 excludes the second, and vice versa. In other words, as the insured Esperanza
Esperanza J. Villanueva survived the insurance period, for she died only on was living on April 1 and March 31, 1943, the proceeds are payable exclusively
October 15, 1944, without, however, collecting the insurance proceeds. to her or to her estate unless she had before her death otherwise assigned the
CFI: estate of the insured Esperanza is entitled to the insurance proceeds matured policies.
ISSUE: W/N the estate of insured Esperanza should be entitled to the insurance
proceeds since she outlived the insurance policy The beneficiary could be entitled to said proceeds only in default of the first
contingency. To sustain the beneficiary’s claim would be to altogether eliminate
from the policies the condition that the insurer “agrees to pay to the insured if
HELD: YES. appealed order is, therefore, hereby affirmed living.”
To sustain the beneficiary's claim would be altogether eliminate from the policies
the condition that the insurer "agrees to pay . . . to the insured hereunder, if This conclusion tallies with American Authorities who say that: The interest of
living the insured in the proceeds of the insurance depends upon his survival of the
Upon the insured's death, within the period, the beneficiary will take, as against expiration of the endowment period. Upon the insured’s death, within the period,
the personal representative or the assignee of the insured. Upon the other hand, the beneficiary will take, as against the personal representatives the endowment
if the insured survives the endowment period, the benefits are payable to him or period, the benefits are payable to him or to his assignee, notwithstanding a
to his assignee, notwithstanding a beneficiary is designated in the policy beneficiary is designated in the policy. (AmJur and Couch Cyclopedia of
Insurance Law)
Villanueva v. Oro - Insurance Proceeds
81 PHIL 464 WEEK 6
Facts:
Harvardian Colleges v. Country Bankers Insurance Corp.
1 CARA 2
Page 35 of 47
Facts: Lessons Applicable: Existing Interest (Insurance)
> Harvardian is a family corporation, the stockholders of which are Ildefonso Laws Applicable: Article 1504,Article 1263, Article 2207 of the Civil
Yap, Virginia King Yap and their children. Code, Section 13 of Insurance Code
> Prior to Aug. 9, 1979, an agent of Country Bankers proposed to Harvardian
to insure its school building. Although at first reluctant, Harvardian agreed.
> Country Banks sent an inspector to inspect the school building and agreed to FACTS:
insure the same for P500,000 for which Harvardian paid an annual premium of Intercapitol Marketing Corporation (IMC) is the maker of Wrangler Blue Jeans.
P2,500. while Levi Strauss (Phils.) Inc. (LSPI) is the local distributor of products bearing
> On Aug. 9, 1979, Country Bankers issued to Harvardian a fire insurance trademarks owned by Levi Strauss & Co
policy. On March 12, 1980, (39 days before I was born… hehehehe )during the IMC and LSPI separately obtained from Insurance Company of North America fire
effectivity of said insurance policy, the insured property was totally burned insurance policies for their book debt endorsements related to their ready-made
rendering it a total loss. clothing materials which have been sold or delivered to various customers and
> A claim was made by plaintiff upon defendant but defendant denied it dealers of the Insured anywhere in the Philippines which are unpaid 45 days after
contending that plaintiff had no insurable interest over the building constructed the time of the loss
on the piece of land in the name of the late Ildefonso Yap as owner. February 25, 1991: Gaisano Superstore Complex in Cagayan de Oro City, owned
> It was contended that both the lot and the building were owned by Ildefonso by Gaisano Cagayan, Inc., containing the ready-made clothing materials sold
Yap and NOT by the Harvardian Colleges. and delivered by IMC and LSPI was consumed by fire.
February 4, 1992: Insurance Company of North America filed a complaint for
Issue: damages against Gaisano Cagayan, Inc. alleges that IMC and LSPI filed their
Whether or not Harvardian colleges has a right to the proceeds. claims under their respective fire insurance policies which it paid thus it was
subrogated to their rights
Held: Gaisano Cagayan, Inc: not be held liable because it was destroyed due to
Harvardian has a right to the proceeds. fortuities event or force majeure
Regardless of the nature of the title of the insured or even if he did not have title RTC: IMC and LSPI retained ownership of the delivered goods until fully paid, it
to the property insured, the contract of fire insurance should still be upheld if his must bear the loss (res perit domino)
interest in or his relation to the property is such that he will be benefited in its CA: Reversed - sales invoices is an exception under Article 1504 (1) of the Civil
continued existence or suffer a direct pecuniary loss from its destruction or Code to res perit domino
injury. The test in determining insurable interest in property is whether one will ISSUE: W/N Insurance Company of North America can claim against Gaisano
derive pecuniary benefit or advantage from its preservation, or will suffer Cagayan for the debt that was isnured
pecuniary loss or damage from its destruction, termination or injury by the
happening of the event insured against. HELD: YES. petition is partly GRANTED. order to pay P535,613 is DELETED
insurance policy is clear that the subject of the insurance is the book debts and
Here Harvardian was not only in possession of the building but was in fact using NOT goods sold and delivered to the customers and dealers of the insured
the same for several years with the knowledge and consent of Ildefonso Yap. It ART. 1504. Unless otherwise agreed, the goods remain at the seller's risk until
is reasonably fair to assume that had the building not been burned, Harvardian the ownership therein is transferred to the buyer, but when the ownership
would have been allowed the continued use of the same as the site of its therein is transferred to the buyer the goods are at the buyer's risk whether
operation as an educational institution. Harvardian therefore would have been actual delivery has been made or not, except that:
directly benefited by the preservation of the property, and certainly suffered a
pecuniary loss by its being burned. (1) Where delivery of the goods has been made to the buyer or to a bailee for
the buyer, in pursuance of the contract and the ownership in the goods has been
retained by the seller merely to secure performance by the buyer of his
Gaisano Cagayan, Inc. V. Insurance Company Of North America (2006) obligations under the contract, the goods are at the buyer's risk from the time
of such delivery;
G.R. No. 147839 June 8, 2006
Page 36 of 47
IMC and LSPI did not lose complete interest over the goods. They have an J. Martinez
insurable interest until full payment of the value of the delivered goods. Unlike
the civil law concept of res perit domino, where ownership is the basis for Facts:
consideration of who bears the risk of loss, in property insurance, one's interest IMC and Levi Strauss (Phils.) Inc. (LSPI) separately obtained from respondent
is not determined by concept of title, but whether insured has substantial fire insurance policies with book debt endorsements. The insurance policies
economic interest in the property provide for coverage on "book debts in connection with ready-made clothing
Section 13 of our Insurance Code defines insurable interest as "every interest in materials which have been sold or delivered to various customers and dealers of
property, whether real or personal, or any relation thereto, or liability in respect the Insured anywhere in the Philippines."
thereof, of such nature that a contemplated peril might directly damnify the The policies defined book debts as the "unpaid account still appearing in the Book
insured." Parenthetically, under Section 14 of the same Code, an insurable of Account of the Insured 45 days after the time of the loss covered under this
interest in property may consist in: (a) an existing interest; (b) an inchoate Policy." The policies also provide for the following conditions:
interest founded on existing interest; or (c) an expectancy, coupled with an 1. Warranted that the Company shall not be liable for any unpaid account in
existing interest in that out of which the expectancy arises. respect of the merchandise sold and delivered by the Insured which are
Anyone has an insurable interest in property who derives a benefit from its outstanding at the date of loss for a period in excess of six (6) months from the
existence or would suffer loss from its destruction. date of the covering invoice or actual delivery of the merchandise whichever shall
it is sufficient that the insured is so situated with reference to the property that first occur.
he would be liable to loss should it be injured or destroyed by the peril against 2. Warranted that the Insured shall submit to the Company within twelve (12)
which it is insured days after the close of every calendar month all amount shown in their books of
an insurable interest in property does not necessarily imply a property interest accounts as unpaid and thus become receivable item from their customers and
in, or a lien upon, or possession of, the subject dealers.
matter of the insurance, and neither the title nor a beneficial interest is requisite Gaisano is a customer and dealer of the products of IMC and LSPI. On February
to the existence of such an interest 25, 1991, the Gaisano Superstore Complex in Cagayan de Oro City, owned by
insurance in this case is not for loss of goods by fire but for petitioner's accounts petitioner, was consumed by fire. Included in the items lost or destroyed in the
with IMC and LSPI that remained unpaid 45 days after the fire - obligation is fire were stocks of ready-made clothing materials sold and delivered by IMC and
pecuniary in nature LSPI.
obligor should be held exempt from liability when the loss occurs thru a fortuitous Insurance of America filed a complaint for damages against Gaisano. It alleges
event only holds true when the obligation consists in the delivery of a that IMC and LSPI were paid for their claims and that the unpaid accounts of
determinate thing and there is no stipulation holding him liable even in case of petitioner on the sale and delivery of ready-made clothing materials with IMC
fortuitous event was P2,119,205.00 while with LSPI it was P535,613.00.
Article 1263 of the Civil Code in an obligation to deliver a generic thing, the loss The RTC rendered its decision dismissing Insurance's complaint. It held that the
or destruction of anything of the same kind does not extinguish the obligation fire was purely accidental; that the cause of the fire was not attributable to the
(Genus nunquan perit) negligence of the petitioner. Also, it said that IMC and LSPI retained ownership
The subrogation receipt, by itself, is sufficient to establish not only the of the delivered goods and must bear the loss.
relationship of respondent as insurer and IMC as the insured, but also the amount The CA rendered its decision and set aside the decision of the RTC. It ordered
paid to settle the insurance claim Gaisano to pay Insurance the P 2 million and the P 500,000 the latter paid to
Art. 2207. If the plaintiff's property has been insured, and he has received IMC and Levi Strauss.
indemnity from the insurance company for the injury or loss arising out of the Hence this petition.
wrong or breach of contract complained of, the insurance company shall be
subrogated to the rights of the insured against the wrongdoer or the person who Issues:
has violated the contract. 1. WON the CA erred in construing a fire insurance policy on book debts as one
As to LSPI, no subrogation receipt was offered in evidence. covering the unpaid accounts of IMC and LSPI since such insurance applies to
Failure to substantiate the claim of subrogation is fatal to petitioner's case for loss of the ready-made clothing materials sold and delivered to petitioner
recovery of the amount of P535,613 2. WON IMC bears the risk of loss because it expressly reserved ownership of
the goods by stipulating in the sales invoices that "[i]t is further agreed that
Gaisano v Insurance G.R. No. 147839 June 8, 2006 merely for purpose of securing the payment of the purchase price the above
Page 37 of 47
described merchandise remains the property of the vendor until the purchase therein, in other words, so long as he would suffer by its destruction, as where
price thereof is fully paid." he has a vendor's lien. In this case, the insurable interest of IMC and LSPI pertain
3. WON petitioner is liable for the unpaid accounts to the unpaid accounts appearing in their Books of Account 45 days after the
4. WON it has been established that petitioner has outstanding accounts with time of the loss covered by the policies.
IMC and LSPI. 3. Petitioner's argument that it is not liable because the fire is a fortuitous event
under Article 117432 of the Civil Code is misplaced. As held earlier,
Held: No. Yes. Yes. Yes but account with LSPI unsubstantiated. Petition partly petitioner bears the loss under Article 1504 (1) of the Civil Code.
granted. Moreover, it must be stressed that the insurance in this case is not for loss of
goods by fire but for petitioner's accounts with IMC and LSPI that remained
Ratio: unpaid 45 days after the fire. Accordingly, petitioner's obligation is for the
1. Nowhere is it provided in the questioned insurance policies that the subject of payment of money. As correctly stated by the CA, where the obligation consists
the insurance is the goods sold and delivered to the customers and dealers of in the payment of money, the failure of the debtor to make the payment even
the insured. by reason of a fortuitous event shall not relieve him of his liability. The rationale
Thus, what were insured against were the accounts of IMC and LSPI with for this is that the rule that an obligor should be held exempt from liability when
petitioner which remained unpaid 45 days after the loss through fire, and not the the loss occurs thru a fortuitous event only holds true when the obligation
loss or destruction of the goods delivered. consists in the delivery of a determinate thing and there is no stipulation holding
2. The present case clearly falls under paragraph (1), Article 1504 of the Civil him liable even in case of fortuitous event. It does not apply when the obligation
Code: is pecuniary in nature.
ART. 1504. Unless otherwise agreed, the goods remain at the seller's risk until Under Article 1263 of the Civil Code, "[i]n an obligation to deliver a generic thing,
the ownership therein is transferred to the buyer, but when the ownership the loss or destruction of anything of the same kind does not extinguish the
therein is transferred to the buyer the goods are at the buyer's risk whether obligation." This rule is based on the principle that the genus of a thing can never
actual delivery has been made or not, except that: perish. An obligation to pay money is generic; therefore, it is not excused by
(1) Where delivery of the goods has been made to the buyer or to a bailee for fortuitous loss of any specific property of the debtor.
the buyer, in pursuance of the contract and the ownership in the goods has been 4. With respect to IMC, the respondent has adequately established its claim. The
retained by the seller merely to secure performance by the buyer of his P 3 m claim has been proven. The subrogation receipt, by itself, is sufficient to
obligations under the contract, the goods are at the buyer's risk from the time establish not only the relationship of respondent as insurer and IMC as the
of such delivery insured, but also the amount paid to settle the insurance claim. The right of
Thus, when the seller retains ownership only to insure that the buyer will pay its subrogation accrues simply upon payment by the insurance company of the
debt, the risk of loss is borne by the buyer. Petitioner bears the risk of loss of insurance claim Respondent's action against petitioner is squarely sanctioned by
the goods delivered. Article 2207 of the Civil Code which provides:
IMC and LSPI had an insurable interest until full payment of the value of the Art. 2207. If the plaintiff's property has been insured, and he has received
delivered goods. Unlike the civil law concept of res perit domino, where indemnity from the insurance company for the injury or loss arising out of the
ownership is the basis for consideration of who bears the risk of loss, in property wrong or breach of contract complained of, the insurance company shall be
insurance, one's interest is not determined by concept of title, but whether subrogated to the rights of the insured against the wrongdoer or the person who
insured has substantial economic interest in the property. has violated the contract.
Section 13 of our Insurance Code defines insurable interest as "every interest in As to LSPI, respondent failed to present sufficient evidence to prove its cause of
property, whether real or personal, or any relation thereto, or liability in respect action. There was no evidence that respondent has been subrogated to any right
thereof, of such nature that a contemplated peril might directly damnify the which LSPI may have against petitioner. Failure to substantiate the claim of
insured." Parenthetically, under Section 14 of the same Code, an insurable subrogation is fatal to petitioner's case for recovery of P535,613.00.
interest in property may consist in: (a) an existing interest; (b) an inchoate
interest founded on existing interest; or (c) an expectancy, coupled with an Harding v Commerical Union August 10, 1918 G.R. No. L-12707
existing interest in that out of which the expectancy arises. J. Fisher
Anyone has an insurable interest in property who derives a benefit from its
existence or would suffer loss from its destruction. Indeed, a vendor or seller Facts:
retains an insurable interest in the property sold so long as he has any interest
Page 38 of 47
Smith Bell insured Mrs. Hardings’ Studebaker car for a premium of Php 150. It was insured Under these circumstances, we do not think that the facts stated in the proposal can be held
for Php 3,000, the value of the car. The car was destroyed by fire. Mrs. Harding furnished as a warranty of the insured, even if it should have been shown that they were incorrect in
the defendant the proofs of her loss, but the company didn’t pay. Evidence showed that the absence of proof of willful misstatement. Under such circumstance, the proposal is to be
Hermanos sold the automobile to Canson for P3,200. Canson then sold the car to Harding regarded as the act of the insurer and not of the insured.
for Php 1,500. The car was then sold for P2,000. It was then resold to Harding. He gave the The defendant, upon the information given by plaintiff, and after an inspection of the
car to his wife; Mrs. Henry E. Harding as a present. The automobile was repaired and automobile by its examiner, having agreed that it was worth P3,000, is bound by this
repainted at the Luneta Garage at P900. valuation in the absence of fraud on the part of the insured. All statements of value are, of
The company averred that they gave false information, particularly that on the price of the necessity, to a large extent matters of opinion, and it would be outrageous to hold that the
vehicle and the ownership of the car. Hence, they aimed to declare the policy void. validity of all valued policies must depend upon the absolute correctness of such estimated
The trial court found that there was no fraud. value.
This was an action by plaintiffs to recover from defendant the sum of P3,000 and interest, Supreme Court v First National- The ordinary test of the value of property is the price it will
alleged to be due under the terms of a policy of insurance. The trial court gave plaintiffs commend in the market if offered for sale. But that test cannot, in the very nature of the case,
judgment for the amount demanded, with interest and costs, and from that decision the be applied at the time application is made for insurance. Men may honestly differ about the
defendant appealed. value of property, or as to what it will bring in the market; and such differences are often very
marked among those whose special business it is to buy and sell property of all kinds.
Issue: Was the valuation of the car for P3000 done fraudulently, thereby making the policy The assured could do no more than estimate such value; and that, it seems, was all that he
void? was required to do in this case. His duty was to deal fairly with the Company in making such
estimate.
Held: No. Section 163 of the Insurance Law (Act No. 2427) provides that “the effect of a valuation in a
policy of fire insurance is the same as in a policy of marine insurance.”
Ratio: By the terms of section 149 of the Act cited, the valuation in a policy of marine insurance is
The policy stated that conclusive if the insured had an insurable interest and was not guilty of fraud.
“That during the period above set forth and during any period for which the company may The valuation of the automobile, for the purposes of the insurance, is binding upon the
agree to renew this policy the company will subject to the exception and conditions contained defendant corporation.
herein or endorsed hereon indemnify the insured against loss of or damage to any motor car
described in the schedule by whatever cause such loss or damage may be occasioned and
will further indemnify the insured up to the value of the car or P3,000 whichever is the greater
against any claim at common law made by any person for loss of life or for accidental bodily Insurance Case Digest: Lampano V. Jose (1915)
injury or damage to property caused by the said motor car including law costs payable in
connection with such claim when incurred with the consent of the company.”
Defendant contends that the statement regarding the cost of the automobile was a warranty, G.R. No. L-9401 March 30, 1915
that the statement was false, and that, therefore, the policy never attached to the risk. Lessons Applicable: Existing Interest (Insurance)
The automobile had in fact cost more than the amount mentioned. The court below found,
and the evidence shows, that the automobile was bought by plaintiff’s husband a few weeks Laws Applicable:
before the issuance of the policy in question for the sum of P2,800, and that between that
time and the issuance of the policy some P900 was spent upon it in repairs and repainting. FACTS:
The mechanic who testified told that the automobile was practically as good as new at the
time the insurance was effected.
The amount stated was less than the actual outlay which the automobile represented to Mr.  Mariano R. Barretto, constructed a house for Placida A. Jose sold the house
Harding, including repairs, when the insurance policy was issued. It would be unfair to hold to Antonina Lampano for P6,000
the policy void simply because the outlay represented by the automobile was made by the
plaintiff’s husband and not by his wife, to whom he had given the automobile.  The house was destroyed by fire during which Lampano still owed Jose
The trial court found that Mrs. Harding, in fixing the value of the automobile at P3,000, acted P2,000 as evidenced by a promissory note. Jose also owed Barretto P2,000
upon information given her by her husband and by Mr. Server, the manager of the Luneta
for the construction.
Garage. She merely repeated the information which had been given her by her husband,
and at the same time disclosed to defendant’s agent the source of her information. There is  After the completion of the house and before it was destroyed, Mariano R. Barretto took
no evidence to sustain the contention that this communication was made in bad faith. out an insurance policy upon it in his own name, with the consent of Placida A. Jose, for

Page 39 of 47
the sum of P4,000. After its destruction, he collected P3,600 from the insurance company,  At the same date, Vicente Ong Lim Sing, Jr. (Lim) an executed an Individual
Guaranty Agreement with FEB to guarantee the prompt and faithful
having paid in premiums the sum of P301.50 performance of the terms and conditions of the lease agreement
 Lampano filed a complaint against Barreto and Jose alleging that Jose in a verbal  JVL defaulted in the payment of the monthly rentals resulting to arrears
of P3,414,468.75 and refused to pay despite demands
agreement told her that the policy will be delivered to her so she should collected P3,600
 FEB filed a complaint for damages and replevin against JVL, Lim and John
from each of them Doe
 RTC: favored Jose ordering Barreto to pay him P1,298.50 and offsetting the  JVL and Lim admitted the existence of the lease agreement but asserted that it is in reality
a sale of equipment on installment basis, with FEB acting as the financier
P2,000  RTC: Sale on installment and the FEB elected full payment of the obligation
 Barreto alone appealed so for the unreturned units and machineries the JVL and Lim are jointly and
ISSUE: W/N Barreto had insurable interest in the house and could insure it for severally liable to pay
 CA: granted FEB appeal that it is a financial lease agreement under Republic
his it for his own protection Act (R.A.) No. 8556 and ordered JVL and Lim jointly and severally to
pay P3,414,468.75
ISSUE: W/N JVL and Lim should jointly and severally be liable for the insured
HELD: YES. reversed and Barretto is absolved
financial lease

 Where different persons have different interests in the same property, the insurance HELD: YES. CA affirmed.

taken by one in his own right and in his own interest does not in any way insure to the  contract of adhesion is as binding as any ordinary contract
 The Lease Contract with corresponding Lease Schedules with Delivery and Acceptance
benefit of another
Certificates is, in point of fact, a financial lease within the purview of R.A. No. 8556
 A contract of insurance made for the insurer's (insured) indemnity only, as where there is  FEB leased the subject equipment and motor vehicles to JVL in consideration of a monthly
no agreement, express or implied, that it shall be for the benefit of a third person, does not periodic payment of P170,494.00. The periodic payment by petitioner is sufficient to
amortize at least 70% of the purchase price or acquisition cost of the said movables in
attach to or run with the title to the insured property on a transfer thereof personal as accordance with the Lease Schedules with Delivery and Acceptance Certificates.
between the insurer and the insured.  JVL entered into the lease contract with full knowledge of its terms and conditions.
 Lim, as a lessee, has an insurable interest in the equipment and motor vehicles leased.
 Barretto had an insurable interest in the house. He construed the building, furnishing all
 In the financial lease agreement, FEB did not assume responsibility as to the quality,
the materials and supplies, and insured it after it had been completed merchantability, or capacity of the equipment. This stipulation provides that, in case of
defect of any kind that will be found by the lessee in any of the equipment, recourse
Insurance Case Digest: Ong Lim Sing V. FEB Leasing And Finance Corp. (2007) should be made to the manufacturer. “The financial lessor, being a financing company, i.e.,
an extender of credit rather than an ordinary equipment rental company, does not extend
a warranty of the fitness of the equipment for any particular use. Thus, the financial
G.R. No. 168115 June 8, 2007
lessee was precisely in a position to enforce such warranty directly against the supplier of
the equipment and not against the financial lessor. We find nothing contra legem or
Lessons Applicable: Existing Interest (Insurance) contrary to public policy in such a contractual arrangement

Laws Applicable:
Insurance Case Digest: Traders Insurance & Surety Co. V. Golangco, Et Al (1954)
FACTS:
G.R. No. L-6442 Sep 21, 1954
 FEB Leasing and Finance Corporation (FEB) leased equipment and motor vehicles to JVL
Lessons Applicable: Existing Interest (Insurance)
Food Products with a monthly rental of P170,494
Laws Applicable: Sec. 13 of the Insurance Code

FACTS:
Page 40 of 47
 Tomas Lianco and the Archbishop entered into a contract of lease on a substantially complied with by Golangco when he made a full and clear
parcel of landowned by church statement of his interests to Trader's manager.
 As lessee, Lianco erected a building on the leased portion of the church’s  The contract between Lianco and the Archbishop only forbade Lianco from
land. transferring 'his rights as LESSEE but the contracts Lianco made in favor of
 Lianco transferred ownership of this building to Kaw Eng Kaw Eng Siand plaintiff Golangco did not transfer such rights; hence no
Si,who later transferred the same to Golangco. written consent thereto was necessary. At worst, the contract would be
 Transfers were made without the consent of the Archbishop voidable, but not a void contract, at the
 The Archbishop filed an ejectment case against Lianco, who appears to be option of the Archbishop and it does not appear that it was ever exercised
occupants of the premises building with others paying rent to Golangco.
 The right of Golangco to receive rent on the building was judicially
recognized in a case decided between Lianco and
others occupying the premises pursuant to a compromise agreement.
 The Archbishop did not exercise his option to question Golangco’s rights as
lessee Insurance Case Digest: Lopez V. Del Rosario And Quiogue (1922)
 April 7,1949: Golangco applied for fire insurance with Trader’s
Insurance and Surety Co.
 fire insurance policy states: "that all insurancecovered under said policy,
includes the 'rent or G.R. No. L-19189 November 27, 1922
othersubject matter of insurance in respect of or inconnection with any
building or any property contained in any building"
 June 5, 1949: the building premises was burned so Golangco requested Lessons Applicable: Carrier or Depositary (Insurance)
Trader’s Insurance to pay the insurance amount of 10,000 including the
amount of rent P1,100 monthly. Laws Applicable:
 Trader’s insurance refused to pay the insurance for the rent averring that
Golangco has no insurable interest
ISSUE: W/N Golangco has insurable interest FACTS:

on the rent of the building premises which may lawfully/validly be subject  Benita Quiogue de V. del Rosario (Mrs. del Rosario), owner of a bonded warehouse
of insurance? where Froilan Lopez, holder or 14 waehouse receipts and Elias Zamora had

HELD: YES. their copra deposited


 The warehouse recipts states an insurance of 1% their declared value which
 Sec. 13 of the Insurance Code:
Every interest in the property, whether real or personal, or any relation can be increase or decrease by giving 1 month's notice in writing
thereto, or liability in respect thereof of such nature that a contemplated  Lopez paid the insurance to May 18, 1920, but not thereafter
peril might directly damnify the insured, is an insurable interest.
 June 6, 1920: the warehouse was destroyed by fire. Only copra worth P49,985 was
 Both at the time of the issuance of the policy and at the time of the fire,
Golangco was in legal possession of the premises, collecting rentals from its salvaged. At that time Lopez was still liable for the storage and insurance
occupant. of P315.90
 The argument of Trader’s Insurance that a policy of insurance must specify
 Mrs. Del Rosario submitted the insurance with the arbitrators and seems to have satisfied
the interest of the insured in the property insured, if he is not the absolute
owner thereof, is not meritorious because it was the Trader’s, not all of the persons who had copra stored in her warehouse, including the stockholders in
Golangco, who prepared that policy, and it cannot take advantage of its the Compañia Coprera de Tayabas (whose stock she took over), with the exception of
own acts to plaintiff's detriment; and, in any case, this provisionwas
Froilan Lopez

Page 41 of 47
 Ineffectual attempts by Mrs. Del Rosario to effect a compromise with Lopez first for requested the Brewery Company to procure insurance on the property in the amount of
P15,000 at Dunn's expense.
P71,994, later raised to P72,724, and finally reduced to P17,000, were made. But Lopez  San Miguel insured the property only as mortgagee.
stubbornly contended, or, at least, his attorney contended for him, that he should receive  Dunn sold the propert to Henry Harding. The insurance was not assigned by Dunn to
Harding.
not a centavo less than P88,595.43 (from originally P107,990.40)  When it was destroyed by fire, the two companies settled with San Miguelto
Issue: the extent of the mortgage credit.
 RTC: Absolved the 2 companies from the difference. Henry Harding is
Whether or not Del Rosario acted as the agent of Lopez in taking out the insurance on the not entitled to the difference between the mortgage credit and the face
contents of the warehouse or whether she acted as the reinsurer of the copra. value of the policies.
 Henry Harding appealed.
Held:
ISSUE:
She acted as the agent of Lopez. 1. W/N San Miguel has insurable interest as mortgagor only to the extent of
the mortgage credit - YES
The agency can be deduced from the warehouse receipts, the insurance policies and the 2. W/N Harding has insurable interest as owner - NO
circumstances surrounding the transaction. Under any aspect, Del Rosario is liable. The
law is that a policy effected by a bailee and covering by its terms in his own property and
property held in trust, inures, in the event of loss, equally and proportionately to the benefit HELD: affirmed
of all owners of the property insured. Even if one secured insurance covering his own
goods and goods stored with him, and even if the owner of the stored goods did not  section 19 of the Insurance Act:
request or know the insurance, and did not ratify it before the payment of the loss, it has  a change of interest in any part of a thing insured unaccompanied by a
been held by a reputable court that the warehouseman is liable to the owner of such stored corresponding change of interest in the insurance, suspends the insurance
goods for his share. to an equivalent extent, until the interest in the thing and the interest in
the insurance are vested in the same person
In a case of contributing policies, adjustments of loss made by an expert or by a board of  section 55:
arbitrators may be submitted to the court NOT as evidence of the facts stated therein, or as  the mere transfer of a thing insured does not transfer the policy, but suspends it until the
obligatory, but for the purpose of assisting the court in calculating the amount of liability. same person becomes the owner of both the policy and the thing insured
Insurance Case Digest: San Miguel Brewery V. Law Union And Rock Insurance Co.  Undoubtedly these policies of insurance might have been so framed as to
(1920) have been "payable to the San Miguel Brewery, mortgagee, as its interest
may appear, remainder to whomsoever, during the continuance of the risk,
G.R. No. L-14300 January 19, 1920 may become the owner of the interest insured." (Sec 54, Act No. 2427.)
Such a clause would have proved an intention to insure the entire interest
Lessons Applicable: in the property, not merely the insurable interest of the San Miguel
 Mortgagor (Insurance) Brewery, and would have shown exactly to whom the money, in case of
 Measure of Insurable Interest (Insurance) loss, should be paid. But the policies are not so written.
 Effect of Change of Interest in Thing Insured (Insurance)  The blame for the situation thus created rests, however, with the Brewery rather than
 Effect of transfer of thing insured (Insurance) with the insurance companies, and there is nothing in the record to indicate that the
Laws Applicable: sec. 16,sec. 19 (now sec. 20),sec. 50,sec.55 (now sec. 58) of insurance companies were requested to write insurance upon the insurable interest of the
the Insurance Code (all old law) owner or intended to make themselves liable to that extent
FACTS:  If by inadvertence, accident, or mistake the terms of the contract were not
fully set forth in the policy, the parties are entitled to have it reformed. But
 In the contract of mortgage, the owner P.D. Dunn had agreed, at his own expense, to to justify the reformation of a contract, the proof must be of the most
insure the mortgaged property for its full value and to indorse the policies in such manner satisfactory character, and it must clearly appear that the contract failed to
as to authorize the Brewery Company to receive the proceeds in case of loss and to retain express the real agreement between the parties
such part thereof as might be necessary to satisfy the remainder then due upon the  In the case now before us the proof is entirely insufficient to authorize reformation.
mortgage debt. Instead, however, of effecting the insurance himself Dunn authorized and

Page 42 of 47
the contract of insurance is a mere wager which is void under Section 25 of
the Insurance Code.
 SECTION 25. Every stipulation in a policy of Insurance for the payment of
Insurance Case Digest: Cha V. CA (1997) loss, whether the person insured has or has not any interest in the property
insured, or that the policy shall be received as proof of such interest, and
every policy executed by way of gaming or wagering, is void
G.R. No. 124520 August 18, 1997
 Section 17. The measure of an insurable interest in property is the extent
Lessons Applicable: Effect of Lack of Insurable Interest (Insurance) to which the insured might be damnified by loss of injury thereof
 The automatic assignment of the policy to CKS under the provision of the
Laws Applicable: Sec. 17, Sec. 18, Sec. 25 of the Insurance Code lease contract previously quoted is void for being contrary to law and/or
public policy. The proceeds of the fire insurance policy thus rightfully
FACTS: belong to the spouses. The liability of the Cha spouses to CKS for violating
their lease contract in that Cha spouses obtained a fire insurance policy
 Spouses Nilo Cha and Stella Uy-Cha and CKS Development Corporation over their own merchandise, without the consent of CKS, is a separate and
entered a 1 year lease contract with a stipulation not to insure against fire distinct issue which we do not resolve in this case.
the chattels, merchandise, textiles, goods and effects placed at any stall or
store or space in the leased premises without first obtaining the written
consent and approval of the lessor. But it insured against loss by fire their Insurance Case Digest: Tai Tong Chuache & Co. V. Insurance Commission (1988)
merchandise inside the leased premises for P500,000 with the United
Insurance Co., Inc. without the written consent of CKS
 On the day the lease contract was to expire, fire broke out inside the leased G.R. No. L-55397 February 29, 1988
premises and CKS learning that the spouses procured an insurance wrote to
United to have the proceeds be paid directly to them. But United refused so
CKS filed against Spouses Cha and United. Lessons Applicable: When Insurable Interest Must Exist (Insurance)
 RTC: United to pay CKS the amount of P335,063.11 and Spouses Cha to
pay P50,000 as exemplary damages, P20,000 as attorney’s fees and costs Laws Applicable:
of suit
 CA: deleted exemplary damages and attorney’s fees
ISSUE: W/N the CKS has insurable interest because the spouses Cha violated
the stipulation FACTS:
HELD: NO. CA set aside. Awarding the proceeds to spouses Cha.
 Azucena Palomo bought a parcel of land and building from Rolando Gonzales and
 Sec. 18. No contract or policy of insurance on property shall be enforceable assumed a mortgage of the building in favor of S.S.S. which was insured
except for the benefit of some person having an insurable interest in the with S.S.S. Accredited Group of Insurers
property insured  April 19, 1975: Azucena Palomo obtained a loan from Tai Tong Chuache Inc.
 A non-life insurance policy such as the fire insurance policy taken by in the amount of P100,000 and to secure it, the land and building was
petitioner-spouses over their merchandise is primarily a contract of mortgaged
indemnity. Insurable interest in the property insured must exist a t the  June 11, 1975: Pedro Palomo secured a Fire Insurance Policy covering the building for
time the insurance takes effect and at the time the loss occurs. The basis P50,000 with Zenith Insurance Corporation
of such requirement of insurable interest in property insured is based on  July 16, 1975: another Fire Insurance policy was procured from Philippine
sound public policy: to prevent a person from taking out an insurance British Assurance Company, covering the same building for P50,000 and
policy on property upon which he has no insurable interest and collecting the contents thereof for P70,000
the proceeds of said policy in case of loss of the property. In such a case,  Before the occurrence of the peril insured against the Palomos had already paid their
credit due the

Page 43 of 47
 July 31, 1975: building and the contents were totally razed by fire Laws Applicable: sec. 16,sec. 19 (now sec. 20),sec. 50,sec.55 (now sec. 58) of
 Palomo was able to claim P41,546.79 from Philippine British Assurance Co., the Insurance Code (all old law)
P11,877.14 from Zenith Insurance Corporation and P5,936.57 from S.S.S. FACTS:
Group of Accredited Insurers but Travellers Multi-Indemnity refused so it
demanded the balance from the other three but they refused so they filed  In the contract of mortgage, the owner P.D. Dunn had agreed, at his own expense, to
insure the mortgaged property for its full value and to indorse the policies in such manner
against them
as to authorize the Brewery Company to receive the proceeds in case of loss and to retain
 Insurance Commission, CFI: absolved Travellers on the basis that Arsenio
such part thereof as might be necessary to satisfy the remainder then due upon the
Cua was claiming and NOT Tai Tong Chuache
mortgage debt. Instead, however, of effecting the insurance himself Dunn authorized and
 Palomo Appealed
requested the Brewery Company to procure insurance on the property in the amount of
 Travellers reasoned that the policy is endorsed to Arsenio Chua, mortgage creditor P15,000 at Dunn's expense.
 Tai Tong Chuache & Co. filed a complaint in intervention claiming the proceeds of the fire  San Miguel insured the property only as mortgagee.
Insurance Policy issued by travellers  Dunn sold the propert to Henry Harding. The insurance was not assigned by Dunn to
 affirmative defense of lack of insurable interest that before the occurrence of the Harding.
peril insured against the Palomos had already paid their credit due the  When it was destroyed by fire, the two companies settled with San Miguelto
petitioner the extent of the mortgage credit.
ISSUE: W/N Tai Tong Chuache & Co. has insurable interest  RTC: Absolved the 2 companies from the difference. Henry Harding is
not entitled to the difference between the mortgage credit and the face
HELD: YES. Travellers Multi-Indemnity Corporation to pay Tai Tong Chuache & value of the policies.
Co.  Henry Harding appealed.
ISSUE:
 when the creditor is in possession of the document of credit, he need not prove non- 1. W/N San Miguel has insurable interest as mortgagor only to the extent of
payment for it is presumed the mortgage credit - YES
 The validity of the insurance policy taken b petitioner was not assailed by private 2. W/N Harding has insurable interest as owner - NO
respondent. Moreover, petitioner's claim that the loan extended to the Palomos has not HELD: affirmed
yet been paid was corroborated by Azucena Palomo who testified that they are still
 section 19 of the Insurance Act:
indebted to herein petitioner
 a change of interest in any part of a thing insured unaccompanied by a
 Chua being a partner of petitioner Tai Tong Chuache & Company is an agent of the
partnership. Being an agent, it is understood that he acted for and in behalf of the firm corresponding change of interest in the insurance, suspends the insurance
 Upon its failure to prove the allegation of lack of insurable interest on the part of the to an equivalent extent, until the interest in the thing and the interest in
petitioner, Travellers must be held liable the insurance are vested in the same person
 section 55:
 the mere transfer of a thing insured does not transfer the policy, but suspends it until the
same person becomes the owner of both the policy and the thing insured
Insurance Case Digest: San Miguel Brewery V. Law Union And Rock Insurance Co.  Undoubtedly these policies of insurance might have been so framed as to
have been "payable to the San Miguel Brewery, mortgagee, as its interest
may appear, remainder to whomsoever, during the continuance of the risk,
(1920)
may become the owner of the interest insured." (Sec 54, Act No. 2427.)
Such a clause would have proved an intention to insure the entire interest
G.R. No. L-14300 January 19, 1920 in the property, not merely the insurable interest of the San Miguel
Brewery, and would have shown exactly to whom the money, in case of
Lessons Applicable: loss, should be paid. But the policies are not so written.
 The blame for the situation thus created rests, however, with the Brewery rather than
 Mortgagor (Insurance)
with the insurance companies, and there is nothing in the record to indicate that the
 Measure of Insurable Interest (Insurance)
insurance companies were requested to write insurance upon the insurable interest of the
 Effect of Change of Interest in Thing Insured (Insurance)
owner or intended to make themselves liable to that extent
 Effect of transfer of thing insured (Insurance)
Page 44 of 47
 If by inadvertence, accident, or mistake the terms of the contract were not property was stored. If the company intended to rely upon a condition of
fully set forth in the policy, the parties are entitled to have it reformed. But that character, it ought to have been plainly expressed in the policy.
to justify the reformation of a contract, the proof must be of the most  alienation clause - forfeiture if the interest in the property pass from the
satisfactory character, and it must clearly appear that the contract failed to insured
express the real agreement between the parties  there is no alienation within the meaning of the insurance law until the
 In the case now before us the proof is entirely insufficient to authorize reformation. mortgage acquires a right to take possession by default under the terms of
the mortgage. No such right is claimed to have accrued in the case at bar,
and the alienation clause is therefore inapplicable.
 we can not find that there is a preponderance of evidence showing that the
Insurance Case Digest: Bachrach V. British American Assurance Co. (1910) plaintiff did actually set fire or cause fire to be set to the goods in question
 It does not positively appear of record that the automobile in question was
G.R. No. L-5715 December 20, 1910 not included in the other policies. It does appear that the automobile was
saved and was considered as a part of the salvaged. It is alleged that the
Lessons Applicable: Effect of Change of Interest in Thing Insured (Insurance)
salvage amounted to P4,000, including the automobile. This amount
(P4,000) was distributed among the different insurers and the amount of
Laws Applicable:
their responsibility was proportionately reduced. The defendant and
appellant in the present case made no objection at any time in the lower
court to that distribution of the salvage. The claim is now made for the first
FACTS: time.

 E. M. Bachrach insured goods belonging to a general furniture store, such


as iron and brass bedsteads, toilet tables, chairs, ice boxes, bureaus, Argente v West Coast G.R. No. L-24899 March 19, 1928
washstands, mirrors, and sea-grass furniture stored in the ground floor and J. Malcolm
first story of house and dwelling with an authorized agent of the British
American Assurance Company
 British American Assurance Company denied alleging that: Facts:
 property covered by the policy to H. W. Peabody & Co. to secure certain
indebtedness due and owing to said company Bernardo Argente signed an application for joint insurance with his wife in the sum of P2,000.
 interest in certain of the goods covered by the said policy is trasnferred to The wife, Vicenta de Ocampo, signed for the same. All the information contained in the
Macke to secure certain obligations assumed by Macke and on behalf of applications was furnished the agent by Bernardo Argente.
Bachrach Argente was examined by Dr. Sta. Ana, a medical examiner for the West Coast. The result
 willfully placed a gasoline can containing 10 gallons of gasoline close to the was recorded in the Medical Examiner's Report, and with the exception of the signature of
insured goods Bernardo Argente, was in the hand-writing of Doctor Sta. Ana. But the information or answers
 made no proof of the loss with the time required by the condition to the questions contained on the face of the Medical Examiner's Report were furnished the
 RTC: British American Assurance Company liable to bACHRACH doctor by Argente.
ISSUE: W/N Bachrach can claim Vicenta de Ocampo, wife of the plaintiff, was examined at her residence by the same doctor.
The spouses submitted to West Coast Life an amended application, increasing the amount
to P15,000, and asked that the policy be dated May 15, 1925. The amended application was
accompanied by the documents entitled "Short Form Medical Report." In both of these
HELD: YES. lower court affirmed documents appear certain questions and answers.
A temporary policy for P15,000 was issued to Bernardo Argente and his wife as of May 15,
 keeping of inflammable oils on the premises, though prohibited by the but it was not delivered until the first quarterly premium on the policy was paid. More than
policy, does not void it if such keeping is incidental to the business thirty days had elapsed since the applicants were examined. Each of them was required to
 It may be added that there was no provision in the policy prohibiting the file a certificate of health before the policy was delivered.
keeping of paints and varnishes upon the premises where the insured

Page 45 of 47
Vicenta de Ocampo died of cerebral apoplexy. Argente presented a claim in due form to the J. De Castro
West Coast Life Insurance Co. for the payment of the sum of P15,000. It was apparently
disclosed that the answers given by the insured in their medical examinations with regard to
their health were untrue. West Coastrefused to pay the claim and wrote Argente to the effect Facts:
that the claim was rejected due to fraud.
The trial court held the policy null and void, hence this appeal. Ngo Hing filed an application with the Great Pacific for a twenty-year endowment policy in
Issue: WON Argente and Ocampo were guilty of concealment and thereby misled the insurer the amount of P50,000.00 on the life of his one-year old daughter Helen. He supplied the
into accepting the risk? essential data which petitioner Mondragon, the Branch Manager, wrote on the form. The
latter paid the annual premium the sum of P1,077.75 going over to the Company, but he
Held: Yes. Petition dismissed. retained the amount of P1,317.00 as his commission for being a duly authorized agent of
Pacific Life.
Ratio:
Upon the payment of the insurance premium, the binding deposit receipt was issued Ngo
Vicenta de Ocampo, in response to the question asked by the medical examiner, answered
Hing. Likewise, petitioner Mondragon handwrote at the bottom of the back page of the
no to "Have you ever consulted a physician for or have you ever suffered from any ailment
application form his strong recommendation for the approval of the insurance application.
or disease of the brain or nervous system?" She also answered “none” as to the question
Then Mondragon received a letter from Pacific Life disapproving the insurance application.
whether she consumed alcohol of not.
The letter stated that the said life insurance application for 20-year endowment plan is not
To the question, "What physician or physicians, if any, not named above, have you consulted available for minors below seven years old, but Pacific Life can consider the same under the
or been treated by, within the last five years and for what illness or ailment?" she answered Juvenile Triple Action Plan, and advised that if the offer is acceptable, the Juvenile Non-
"None." Medical Declaration be sent to the company.

But the facts show that she was taken to San Lazaro Hospital, her case was diagnosed by The non-acceptance of the insurance plan by Pacific Life was allegedly not communicated
the admitting physician as "alcoholism”, moreover, she was diagnosed with "phycho- by petitioner Mondragon to private respondent Ngo Hing. Instead, on May 6, 1957,
neurosis." Mondragon wrote back Pacific Life again strongly recommending the approval of the 20-year
endowment insurance plan to children, pointing out that since the customers were asking for
Section 25 of the Insurance Code defined concealment as "a neglect to communicate that such coverage.
which a party knows and ought to communicate."
Helen Go died of influenza. Ngo Hing sought the payment of the proceeds of the insurance,
The court held that the alleged concealment was not immaterial and insufficient to avoid the but having failed in his effort, he filed the action for the recovery before the Court of First
policy. In an action on a life insurance policy where the evidence conclusively shows that the Instance of Cebu, which ruled against him.
answers to questions concerning diseases were untrue, the truth of falsity of the answers
become the determining factor. If the true facts been disclosed by the assured, the insurance
would never have been granted.
Issues:
Concealment must, in the absence of inquiries, be not only material, but fraudulent, or the
1. Whether the binding deposit receipt constituted a temporary contract of the life insurance
fact must have been intentionally withheld. If no inquiries are made and no fraud or design
in question
to conceal enters into the concealment the contract is not avoided.
2. Whether Ngo Hing concealed the state of health and physical condition of Helen Go, which
The assurer is entitled to know every material fact of which the assured has exclusive or
rendered void the policy
peculiar knowledge, as well as all material facts which directly tend to increase the hazard
or risk which are known by the assured, or which ought to be or are presumed to be known
by him. And a concealment of such facts vitiates the policy.
Held: No. Yes. Petition dismissed.
If the assured has exclusive knowledge of material facts, he should fully and fairly disclose
the same, whether he believes them material or not. The determination of the point whether Ratio:
there has or has not been a material concealment must rest largely in all cases upon the
The receipt was intended to be merely a provisional insurance contract. Its perfection was
exact terms of the contract.
subject to compliance of the following conditions: (1) that the company shall be satisfied that
Great Pacific v CA G.R. No. L-31845 April 30, 1979 the applicant was insurable on standard rates; (2) that if the company does not accept the
application and offers to issue a policy for a different plan, the insurance contract shall not
Page 46 of 47
be binding until the applicant accepts the policy offered; otherwise, the deposit shall be
refunded; and (3) that if the company disapproves the application, the insurance applied for
shall not be in force at any time, and the premium paid shall be returned to the applicant.
The receipt is merely an acknowledgment that the latter's branch office had received from
the applicant the insurance premium and had accepted the application subject for processing
by the insurance company. There was still approval or rejection the same on the basis of
whether or not the applicant is "insurable on standard rates." Since Pacific Life disapproved
the insurance application of respondent Ngo Hing, the binding deposit receipt in question
had never become in force at any time. The binding deposit receipt is conditional and does
not insure outright. This was held in Lim v Sun.
The deposit paid by private respondent shall have to be refunded by Pacific Life.
2. Ngo Hing had deliberately concealed the state of health of his daughter Helen Go. When
he supplied data, he was fully aware that his one-year old daughter is typically a mongoloid
child. He withheld the fact material to the risk insured.
“The contract of insurance is one of perfect good faith uberrima fides meaning good faith,
absolute and perfect candor or openness and honesty; the absence of any concealment or
demotion, however slight.”
The concealment entitles the insurer to rescind the contract of insurance.

Page 47 of 47

S-ar putea să vă placă și